Neurological System Questions Part 2

You might also like

Download as pdf or txt
Download as pdf or txt
You are on page 1of 35

1. A patient is brought to the ER following a motor vehicle accident in which he sustained head trauma.

Preliminary assessment reveals a


vision deficit in the patients left eye. The nurse should associate this abnormal finding with trauma to which of the following cerebral lobes?
A) Temporal
B) Occipital
C) Parietal
D) Frontal
Ans: B
The posterior lobe of the cerebral hemisphere is responsible for visual interpretation. The temporal lobe contains the auditory receptive areas.
The parietal lobe contains the primary sensory cortex, and is essential to an individuals awareness of the body in space, as well as orientation
in space and spatial relations. The frontal lobe functions in concentration, abstract thought, information storage or memory, and motor fxn

2. A patient scheduled for magnetic resonance imaging (MRI) has arrived at the radiology department. The nurse who prepares the patient for
the MRI should prioritize which of the following actions?
A) Withholding stimulants 24 to 48 hours prior to exam
B) Removing all metal-containing objects
C) Instructing the patient to void prior to the MRI
D) Initiating an IV line for administration of contrast
Ans: B
Patient preparation for an MRI consists of removing all metal-containing objects prior to the examination. Withholding stimulants would not
affect an MRI; this relates to an electroencephalography (EEG). Instructing the patient to void is patient preparation for a lumbar puncture.
Initiating an IV line for administration of contrast would be done if the patient was having a CT scan with contrast.

3. A gerontologic nurse planning the neurologic assessment of an older adult is considering normal, age- related changes. Of what
phenomenon should the nurse be aware?
A) Hyperactive deep tendon reflexes
B) Reduction in cerebral blood flow
C) Increased cerebral metabolism
D) Hypersensitivity to painful stimuli
Ans: B
Reduction in cerebral blood flow (CBF) is a change that occurs in the normal aging process. Deep tendon reflexes can be decreased or, in
some cases, absent. Cerebral metabolism decreases as the patient advances in age. Reaction to painful stimuli may be decreased with age.
Because pain is an important warning signal, caution must be used when hot or cold packs are used.

4. The nurse has admitted a new patient to the unit. One of the patients admitting orders is for an adrenergic medication. The nurse knows that
this medication will have what effect on the circulatory system?
A) Thin, watery saliva
B) Increased heart rate
C) Decreased BP
D) Constricted bronchioles
Ans: B
The term adrenergic refers to the sympathetic nervous system. Sympathetic effects include an increased rate and force of the heartbeat.
Cholinergic effects, which correspond to the parasympathetic division of the autonomic nervous system, include thin, watery saliva,
decreased rate and force of heartbeat, and decreased BP.

5. A nurse is assessing reflexes in a patient with hyperactive reflexes. When the patients foot is abruptly dorsiflexed, it continues to beat two
to three times before settling into a resting position. How would the nurse document this finding?
A) Rigidity
B) Flaccidity
C) Clonus
D) Ataxia
Ans: C
When reflexes are very hyperactive, a phenomenon called clonus may be elicited. If the foot is abruptly dorsiflexed, it may continue to beat
two to three times before it settles into a position of rest. Rigidity is an increase in muscle tone at rest characterized by increased resistance to
passive stretch. Flaccidity is lack of muscle tone. Ataxia is the inability to coordinate muscle movements, resulting in difficulty walking,
talking, and performing self-care activities.

6. The nurse is doing an initial assessment on a patient newly admitted to the unit with a diagnosis of cerebrovascular accident (CVA). The
patient has difficulty copying a figure that the nurse has drawn and is diagnosed with visual-receptive aphasia. What brain region is primarily
involved in this deficit?
A) Temporal lobe
B) Parietal-occipital area
C) Inferior posterior frontal areas
D) Posterior frontal area
Ans: B
Difficulty copying a figure that the nurse has drawn would be considered visual-receptive aphasia, which involves the parietal-occipital area.
Expressive aphasia, the inability to express oneself, is often associated with damage to the frontal area. Receptive aphasia, the inability to
understand what someone else is saying, is often associated with damage to the temporal lobe area.

7. What term is used to describe the fibrous connective tissue that hugs the brain closely and extends into every fold of the brains surface?
A) Dura mater
B) Arachnoid
C) Fascia
D) Pia mater
Ans: D
The term meninges describes the fibrous connective tissue that covers the brain and spinal cord. The meninges have three layers, the dura
mater, arachnoid, and pia mater. The pia mater is the innermost membrane that hugs the brain closely and extends into every fold of the brains
surface. The dura mater, the outermost layer, covers the brain and spinal cord. The arachnoid, the middle membrane, is responsible for the
production of cerebrospinal fluid.

8. The nurse is caring for a patient with an upper motor neuron lesion. What clinical manifestations should the nurse anticipate when planning
the patients neurologic assessment?
A) Decreased muscle tone
B) Flaccid paralysis
C) Loss of voluntary control of movement
D) Slow reflexes
Ans: C
Upper motor neuron lesions do not cause muscle atrophy, flaccid paralysis, or slow reflexes. However, upper motor neuron lesions normally
cause loss of voluntary control.

9. The nurse is admitting a patient to the unit who is diagnosed with a lower motor neuron lesion. What entry in the patients electronic record
is most consistent with this diagnosis?
A) Patient exhibits increased muscle tone.
B) Patient demonstrates normal muscle structure with no evidence of atrophy.
C) Patient demonstrates hyperactive deep tendon reflexes.
D) Patient demonstrates an absence of deep tendon reflexes.
Ans: D
Lower motor neuron lesions cause flaccid muscle paralysis, muscle atrophy, decreased muscle tone, and loss of voluntary control.

10. An elderly patient is being discharged home. The patient lives alone and has atrophy of his olfactory organs. The nurse tells the patients
family that it is essential that the patient have what installed in the home?
A) Grab bars
B) Nonslip mats
C) Baseboard heaters
D) A smoke detector
Ans: D
The sense of smell deteriorates with age. The olfactory organs are responsible for smell. This may present a safety hazard for the patient
because he or she may not smell smoke or gas leaks. Smoke detectors are universally necessary, but especially for this patient.

11. The patient in the ED has just had a diagnostic lumbar puncture. To reduce the incidence of a post- lumbar puncture headache, what is the
nurses most appropriate action?
A) Position the patient prone.
B) Position the patient supine with the head of bed flat.
C) Position the patient left side-lying.
D) Administer acetaminophen as ordered.
Ans: A
The lumbar puncture headache may be avoided if a small-gauge needle is used and if the patient remains prone after the procedure.
Acetaminophen is not administered as a preventative measure for post-lumbar puncture headaches.

12. The nurse is conducting a focused neurologic assessment. When assessing the patients cranial nerve function, the nurse would include
which of the following assessments?
A) Assessment of hand grip
B) Assessment of orientation to person, time, and place
C) Assessment of arm drift
D) Assessment of gag reflex
Ans: D
The gag reflex is governed by the glossopharyngeal nerve, one of the cranial nerves. Hand grip and arm drifting are part of motor function
assessment. Orientation is an assessment parameter related to a mental status examination.

13. A nurse is caring for a patient diagnosed with Mnires disease. While completing a neurologic examination on the patient, the nurse
assesses cranial nerve VIII. The nurse would be correct in identifying the function of this nerve as what?
A) Movement of the tongue
B) Visual acuity
C) Sense of smell
D) Hearing and equilibrium
Ans: D
Cranial nerve VIII (acoustic) is responsible for hearing and equilibrium. Cranial nerve XII (hypoglossal) is responsible for movement of the
tongue. Cranial nerve II (optic) is responsible for visual acuity and visual fields. Cranial nerve I (olfactory) functions in sense of smell.

14. A pt exhibits uncoordinated gait has presented at the clinic. Which of the following is the most plausible cause of this pt’s health problem?
A) Cerebellar dysfunction
B) A lesion in the pons
C) Dysfunction of the medulla
D) A hemorrhage in the midbrain
Ans: A
The cerebellum controls fine movement, balance, position sense, and integration of sensory input. Portions of the pons control the heart,
respiration, and blood pressure. Cranial nerves IX through XII connect to the brain in the medulla. Cranial nerves III and IV originate in the
midbrain.

15. The nursing students are learning how to assess function of cranial nerve VIII. To assess the function of cranial nerve VIII the students
would be correct in completing which of the following assessment techniques?
A) Have the patient identify familiar odors with the eyes closed.
B) Assess papillary reflex.
C) Utilize the Snellen chart.
D) Test for air and bone conduction (Rinne test).
Ans: D
Cranial nerve VIII is the acoustic nerve. It functions in hearing and equilibrium. When assessing this nerve, the nurse would test for air and
bone conduction (Rinne) with a tuning fork. Assessment of papillary reflex would be completed for cranial nerves III (oculomotor), IV
(trochlear), and VI (abducens). The Snellen chart would be used to assess cranial nerve II (optic).

16. A patient is being given a medication that stimulates her parasympathetic system. Following administration of this medication, the nurse
should anticipate what effect?
A) Constricted pupils
B) Dilated bronchioles
C) Decreased peristaltic movement
D) Relaxed muscular walls of the urinary bladder
Ans: A
Parasympathetic stimulation results in constricted pupils, constricted bronchioles, increased peristaltic movement, and contracted muscular
walls of the urinary bladder.

17. A patient with lower back pain is scheduled for myelography using metrizamide (a water-soluble contrast dye). After the test, the nurse
should prioritize what action?
A) Positioning the patient with the head of the bed elevated 45 degrees
B) Administering IV morphine sulfate to prevent headache
C) Limiting fluids for the next 12 hours
D) Helping the patient perform deep breathing and coughing exercises
Ans: A
After myelography, the patient lies in bed with the head of the bed elevated 30 to 45 degrees. The patient is advised to remain in bed in the
recommended position for 3 hours or as prescribed. Drinking liberal amounts of fluid for rehydration and replacement of CSF may decrease
the incidence of postlumbar puncture headache. Deep breathing and coughing exercises are not normally necessary since there is no
consequent risk of atelectasis.

18. A patient is having a fight or flight response after receiving bad news about his prognosis. What affect will this have on the patients
sympathetic nervous system?
A) Constriction of blood vessels in the heart muscle
B) Constriction of bronchioles
C) Increase in the secretion of sweat
D) Constriction of pupils
Ans: C
Sympathetic nervous system stimulation results in dilated blood vessels in the heart and skeletal muscle, dilated bronchioles, increased
secretion of sweat, and dilated pupils.

19. The nurse educator is reviewing the assessment of cranial nerves. What should the educator identify as the specific instances when cranial
nerves should be assessed? Select all that apply.
A) When a neurogenic bladder develops
B) When level of consciousness is decreased
C) With brain stem pathology
D) In the presence of peripheral nervous system disease
E) When a spinal reflex is interrupted
Ans: B, C, D
Cranial nerves are assessed when level of consciousness is decreased, with brain stem pathology, or in the presence of peripheral nervous
system disease. Abnormalities in muscle tone and involuntary movements are less likely to prompt the assessment of cranial nerves, since
these nerves do not directly mediate most aspects of muscle tone and movement.

20. A patient in the OR goes into malignant hyperthermia due to an abnormal reaction to the anesthetic. The nurse knows that the area of the
brain that regulates body temperature is which of the following?
A) Cerebellum
B) Thalamus
C) Hypothalamus
D) Midbrain
Ans: C
The hypothalamus plays an important role in the endocrine system because it regulates the pituitary secretion of hormones that influence
metabolism, reproduction, stress response, and urine production. It works with the pituitary to maintain fluid balance through hormonal
release and maintains temperature regulation by promoting vasoconstriction or vasodilatation. The cerebellum, thalamus, and midbrain and
not directly involved in temperature regulation.

21. The nurse is planning the care of a patient with Parkinsons disease. The nurse should be aware that treatment will focus on what
pathophysiological phenomenon?
A) Premature degradation of acetylcholine
B) Decreased availability of dopamine
C) Insufficient synthesis of epinephrine
D) Delayed reuptake of serotonin
Ans: B
Parkinsons disease develops from decreased availability of dopamine, not acetylcholine, epinephrine, or serotonin.

22. A patient is admitted to the medical unit with an exacerbation of multiple sclerosis. When assessing this patient, the nurse has the patient
stick out her tongue and move it back and forth. What is the nurse assessing?
A) Function of the hypoglossal nerve
B) Function of the vagus nerve
C) Function of the spinal nerve
D) Function of the trochlear nerve
Ans: A
The hypoglossal nerve is the 12th cranial nerve. It is responsible for movement of the tongue. None of the other listed nerves affects motor
function in the tongue.

23. A trauma patient was admitted to the ICU with a brain injury. The patient had a change in level of consciousness, increased V/S, and
became diaphoretic and agitated. The nurse should recognize which of the following syndromes as the most plausible cause of these sx?
A) Adrenal crisis
B) Hypothalamic collapse
C) Sympathetic storm
D) Cranial nerve deficit
Ans: C
Sympathetic storm is a syndrome associated with changes in level of consciousness, altered vital signs, diaphoresis, and agitation that may
result from hypothalamic stimulation of the sympathetic nervous system following traumatic brain injury. Alterations in cranial nerve or
adrenal function would not have this result.

24. Assessment is crucial to care of pt w/neurologic dysfunction. What does accurate & appropriate assessment require? Select all that apply.
A) The ability to select mediations for the neurologic dysfunction
B) Understanding of the tests used to diagnose neurologic disorders
C) Knowledge of nursing interventions related to assessment and diagnostic testing
D) Knowledge of the anatomy of the nervous system
E) The ability to interpret the results of diagnostic tests
Ans: B, C, D
Assessment requires knowledge of the anatomy and physiology of the nervous system and an understanding of the array of tests and
procedures used to diagnose neurologic disorders. Knowledge about the nursing implications and interventions related to assessment and
diagnostic testing is also essential. Selecting medications and interpreting diagnostic tests are beyond the normal scope of the nurse.

25. When caring for a patient with an altered level of consciousness, the nurse is preparing to test cranial nerve VII. What assessment
technique would the nurse use to elicit a response from cranial nerve VII?
A) Palpate trapezius muscle while patient shrugs should against resistance.
B) Administer the whisper or watch-tick test.
C) Observe for facial movement symmetry, such as a smile.
D) Note any hoarseness in the patients voice.
Ans: C
Cranial nerve VII is the facial nerve. An appropriate assessment technique for this cranial nerve would include observing for symmetry while
the patient performs facial movements: smiles, whistles, elevates eyebrows, and frowns. Palpating and noting strength of the trapezius muscle
while the patient shrugs shoulders against resistance would be completed to assess cranial nerve XI (spinal accessory). Assessing cranial
nerve VIII (acoustic) would involve using the whisper or watch-tick test to evaluate hearing. Noting any hoarseness in the patients voice
would involve assessment of cranial nerve X (vagus).

26. The nurse is caring for a patient who exhibits abnormal results of the Weber test and Rinne test. The nurse should suspect dysfunction
involving what cranial nerve?
A) Trigeminal
B) Acoustic
C) Hypoglossal
D) Trochlear
Ans: B
Abnormal hearing can correlate with damage to cranial nerve VIII (acoustic). The acoustic nerve functions in hearing and equilibrium. The
trigeminal nerve functions in facial sensation, corneal reflex, and chewing. The hypoglossal nerve moves the tongue. The trochlear nerve
controls muscles that move the eye.

27. The nurse caring for an 80 year-old patient knows that she has a pre-existing history of dulled tactile sensation. The nurse should first
consider what possible cause for this patients diminished tactile sensation?
A) Damage to cranial nerve VIII
B) Adverse medication effects
C) Age-related neurologic changes
D) An undiagnosed cerebrovascular accident in early adulthood
Ans: C
Tactile sensation is dulled in the elderly person due to a decrease in the number of sensory receptors. While thorough assessment is necessary,
it is possible that this change is unrelated to pathophysiological processes.

28. A 72-year-old man has been brought to his primary care provider by his daughter, who claims that he has been experiencing
uncharacteristic lapses in memory. What principle should underlie the nurses assessment and management of this patient?
A) Loss of short-term memory is normal in older adults, but loss of long-term memory is pathologic.
B) Lapses in memory in older adults are considered benign unless they have negative consequences.
C) Gradual increases in confusion accompany the aging process.
D) Thorough assessment is necessary because changes in cognition are always considered to be
pathologic.
Ans: D
Although mental processing time decreases with age, memory, language, and judgment capacities remain intact. Change in mental status
should never be assumed to be a normal part of aging.

29. A gerontologic nurse educator is providing practice guidelines to unlicensed care providers. Because reaction to painful stimuli is
sometimes blunted in older adults, what must be used with caution?
A) Hot or cold packs
B) Analgesics
C) Anti-inflammatory medications
D) Whirlpool baths
Ans: A
Reaction to painful stimuli may be decreased with age. Because pain is an important warning signal, caution must be used when hot or cold
packs are used. The older pt may be burned or suffer frostbite before being aware of any discomfort. Any medication is used with caution in
elderly, but not b/c of the decreased sense of heat/cold. Whirlpool baths are generally not a routine treatment ordered for the elderly.

30. A trauma patient in the ICU has been declared brain dead. What diagnostic test is used in making the determination of brain death?
A) Magnetic resonance imaging (MRI)
B) Electroencephalography (EEG)
C) Electromyelography (EMG)
D) Computed tomography (CT)
Ans: B
The EEG can be used in determining brain death. MRI, CT, and EMG are not normally used in determining brain death.

31. A patient is scheduled for CT scanning of the head because of a recent onset of neurologic deficits. What should the nurse tell the patient
in preparation for this test?
A) No metal objects can enter the procedure room.
B) You need to fast for 8 hours prior to the test.
C) You will need to lie still throughout the procedure.
D) There will be a lot of noise during the test.
Ans: C
Preparation for CT scanning includes teaching the patient about the need to lie quietly throughout the procedure. If the patient were having an
MRI, metal and noise would be appropriate teaching topics. There is no need to fast prior to a CT scan of the brain.

32. A pt for whom the nurse is caring has positron emission tomography (PET) scheduled. In preparation, what should the nurse explain to pt?
A) The test will temporarily limit blood flow through the brain.
B) An allergy to iodine precludes getting the radio-opaque dye.
C) The patient will need to endure loud noises during the test.
D) The test may result in dizziness or lightheadedness.
Ans: D
Key nursing interventions for PET scan include explaining the test and teaching the patient about inhalation techniques and the sensations
(e.g., dizziness, light-headedness, and headache) that may occur. A PET scan does not impede blood flow through the brain. An allergy to
iodine precludes the dye for an MRI, and loud noise is heard in an MRI.

33. A patient is scheduled for a myelogram and the nurse explains to the patient that this is an invasive procedure, which assesses for any
lesions in the spinal cord. The nurse should explain that the preparation is similar to which of the following neurologic tests?
A) Lumbar puncture
B) MRI
C) Cerebral angiography
D) EEG
Ans: A
A myelogram is an x-ray of the spinal subarachnoid space taken after the injection of a contrast agent into the spinal subarachnoid space
through a lumbar puncture. Patient preparation for a myelogram would be similar to that for lumbar puncture. The other listed diagnostic tests
do not involve lumbar puncture.

34. The physician has ordered a somatosensory evoked responses (SERs) test for a patient for whom the nurse is caring. The nurse is justified
in suspecting that this patient may have a history of what type of neurologic disorder?
A) Hypothalamic disorder
B) Demyelinating disease
C) Brainstem deficit
D) Diabetic neuropathy
Ans: B
SERs are used to detect deficits in the spinal cord or peripheral nerve conduction and to monitor spinal cord function during surgical
procedures. The test is also useful in the diagnosis of demyelinating diseases, such as multiple sclerosis and polyneuropathies, where nerve
conduction is slowed. The test is not done to diagnose hypothalamic disorders, brainstem deficits, or diabetic neuropathies.

35. A patient had a lumbar puncture performed at the outpatient clinic and the nurse has phoned the patient and family that evening. What
does this phone call enable the nurse to determine?
A) What are the patients and familys expectations of the test
B) Whether the patients family had any questions about why the test was necessary
C) Whether the patient has had any complications of the test
D) Whether the patient understood accurately why the test was done
Ans: C
Contacting the pt & family after diagnostic testing enables the nurse to determine whether they have any questions about the procedure or
whether the pt had any untoward results. The other listed information should have been elicited from the patient and family prior to the test.

36. A pt is currently being stimulated by the parasympathetic nervous system. What effect will this nervous stimulation have on the pt’s
bladder?
A) The parasympathetic nervous system causes urinary retention.
B) The parasympathetic nervous system causes bladder spasms.
C) The parasympathetic nervous system causes urge incontinence.
D) The parasympathetic nervous system makes the bladder contract.
Ans: D
The parasympathetic division of the nervous system causes contraction (stimulation) of the urinary bladder muscles and a decrease
(inhibition) in heart rate, whereas the sympathetic division produces relaxation (inhibition) of the urinary bladder and an increase
(stimulation) in the rate and force of the heartbeat.

37. The nurse is performing a neurologic assessment of a patient whose injuries have rendered her unable to follow verbal commands. How
should the nurse proceed with assessing the patients level of consciousness (LOC)?
A) Assess the patients vital signs and correlate these with the patients baselines.
B) Assess the patients eye opening and response to stimuli.
C) Document that the patient currently lacks a level of consciousness.
D) Facilitate diagnostic testing in an effort to obtain objective data.
Ans: B
If the patient is not alert or able to follow commands, the examiner observes for eye opening; verbal response and motor response to stimuli,
if any; and the type of stimuli needed to obtain a response. Vital signs and diagnostic testing are appropriate, but neither will allow the nurse
to gauge the patients LOC. Inability to follow commands does not necessarily denote an absolute lack of consciousness.
38. In the course of a focused neurologic assessment, the nurse is palpating the patients major muscle groups at rest and during passive
movement. Data gleaned from this assessment will allow the nurse to describe which of the following aspects of neurologic function?
A) Muscle dexterity
B) Muscle tone
C) Motor symmetry
D) Deep tendon reflexes
Ans: B
Muscle tone (the tension present in a muscle at rest) is evaluated by palpating various muscle groups at rest and during passive movement.
Data from this assessment do not allow the nurse to ascertain the patients dexterity, reflexes, or motor symmetry.

39. The neurologic nurse is testing the function of a patients cerebellum and basal ganglia. What action will most accurately test these
structures?
A) Have the patient identify the location of a cotton swab on his or her skin with the eyes closed.
B) Elicit the patients response to a hypothetical problem.
C) Ask the patient to close his or her eyes and discern between hot and cold stimuli.
D) Guide the patient through the performance of rapid, alternating movements.
Ans: D
Cerebellar and basal ganglia influence on the motor system is reflected in balance control and coordination. Coordination in the hands and
upper extremities is tested by having the patient perform rapid, alternating movements and point-to-point testing. The cerebellum and basal
ganglia do not mediate cutaneous sensation or judgment.

40. During the performance of the Romberg test, the nurse observes that the pt sways slightly. What is the nurses most appropriate action?
A) Facilitate a referral to a neurologist.
B) Reposition the patient supine to ensure safety.
C) Document successful completion of the assessment.
D) Follow up by having the patient perform the Rinne test.
Ans: C
Slight swaying during the Romberg test is normal, but a loss of balance is abnormal and is considered a positive Romberg test. Slight swaying
is not a significant threat to the patients safety. The Rinne test assesses hearing, not balance.
Chapter 66: Management of Patients with Neurologic Dysfunction
1. A patient is being admitted to the neurologic ICU following an acute head injury that has resulted in cerebral edema. When planning this
patients care, the nurse would expect to administer what priority medication?
A) Hydrochlorothiazide (HydroDIURIL)
B) Furosemide (Lasix)
C) Mannitol (Osmitrol)
D) Spirolactone (Aldactone)
Ans: C
The osmotic diuretic mannitol is given to dehydrate the brain tissue and reduce cerebral edema. This drug acts by reducing the volume of
brain and extracellular fluid. Spirolactone, furosemide, and hydrochlorothiazide are diuretics that are not typically used in the treatment of
increased ICP resulting from cerebral edema.

2. The nurse is providing care for a patient who is unconscious. What nursing intervention takes highest priority?
A) Maintaining accurate records of intake and output
B) Maintaining a patent airway
C) Inserting a nasogastric (NG) tube as ordered
D) Providing appropriate pain control
Ans: B
Maintaining a patent airway always takes top priority, even though each of the other listed actions is necessary and appropriate.

3. The nurse is caring for a patient in the ICU who has a brain stem herniation and who is exhibiting an altered level of consciousness.
Monitoring reveals that the patients mean arterial pressure (MAP) is 60 mm Hg with an intracranial pressure (ICP) reading of 5 mm Hg.
What is the nurses most appropriate action?
A) Position the patient in the high Fowlers position as tolerated.
B) Administer osmotic diuretics as ordered.
C) Participate in interventions to increase cerebral perfusion pressure.
D) Prepare the patient for craniotomy.
Ans: C
The cerebral perfusion pressure (CPP) is 55 mm Hg, which is considered low. The normal CPP is 70 to 100 mm Hg. Patients with a CPP of
less than 50 mm Hg experience irreversible neurologic damage. As a result, interventions are necessary. A craniotomy is not directly
indicated. Diuretics and increased height of bed would exacerbate the patients condition.

4. The nurse is caring for a patient who is postoperative following a craniotomy. When writing the plan of care, the nurse identifies a
diagnosis of deficient fluid volume R/T fluid restriction & osmotic diuretic use. What would be an appropriate intervention for this diagnosis?
A) Change the patients position as indicated.
B) Monitor serum electrolytes.
C) Maintain NPO status.
D) Monitor arterial blood gas (ABG) values.
Ans: B
The postoperative fluid regimen depends on the type of neurosurgical procedure and is determined on an individual basis. The volume and
composition of fluids are adjusted based on daily serum electrolyte values, along with fluid intake and output. Fluids may have to be
restricted in patients with cerebral edema. Changing the patients position, maintaining an NPO status, and monitoring ABG values do not
relate to the nursing diagnosis of deficient fluid volume.

5. A patient with a documented history of seizure disorder experiences a generalized seizure. What nursing action is most appropriate?
A) Restrain the patient to prevent injury.
B) Open the patients jaws to insert an oral airway.
C) Place patient in high Fowlers position.
D) Loosen the patients restrictive clothing.
Ans: D
An appropriate nursing intervention would include loosening any restrictive clothing on the patient. No attempt should be made to restrain the
patient during the seizure because muscular contractions are strong and restraint can produce injury. Do not attempt to pry open jaws that are
clenched in a spasm to insert anything. Broken teeth and injury to the lips and tongue may result from such an action. If possible, place the
patient on one side with head flexed forward, which allows the tongue to fall forward and facilitates drainage of saliva and mucus.

6. A patient who has been on long-term phenytoin (Dilantin) therapy is admitted to the unit. In light of the adverse of effects of this
medication, the nurse should prioritize which of the following in the patients plan of care?
A) Monitoring of pulse oximetry
B) Administration of a low-protein diet
C) Administration of thorough oral hygiene
D) Fluid restriction as ordered
Ans: C
Gingival hyperplasia (swollen and tender gums) can be associated with long-term phenytoin (Dilantin) use. Thorough oral hygiene should be
provided consistently and encouraged after discharge. Fluid and protein restriction are contraindicated and there is no particular need for
constant oxygen saturation monitoring.

7. A nurse is admitting a patient with a severe migraine headache and a history of acute coronary syndrome. What migraine medication would
the nurse question for this patient?
A) Rizatriptan (Maxalt)
B) Naratriptan (Amerge)
C) Sumatriptan succinate (Imitrex)
D) Zolmitriptan (Zomig)
Ans: C
Triptans can cause chest pain and are contraindicated in patients with ischemic heart disease. Maxalt, Amerge, and Zomig are triptans used in
routine clinical use for the treatment of migraine headaches.

8. The nurse is caring for a patient with increased intracranial pressure (ICP). The patient has a nursing diagnosis of ineffective cerebral tissue
perfusion. What would be an expected outcome that the nurse would document for this diagnosis?
A) Copes with sensory deprivation.
B) Registers normal body temperature.
C) Pays attention to grooming.
D) Obeys commands with appropriate motor responses.
Ans: D
An expected outcome of the diagnosis of ineffective cerebral tissue perfusion in a patient with ICP would include obeying commands with
appropriate motor responses. Vitals signs and neurologic status are assessed every 15 minutes to every hour. Coping with sensory deprivation
would relate to the nursing diagnosis of disturbed sensory perception. The outcome of registers normal body temperature R/T the diagnosis of
potential for ineffective thermoregulation. Body image disturbance would have a potential outcome of pays attention to grooming.

9. A patient exhibiting an altered level of consciousness (LOC) due to blunt-force trauma to the head is admitted to the ED. The physician
determines the patients injury is causing increased intracranial pressure (ICP). The nurse should gauge the patients LOC on the results of
what diagnostic tool?
A) Monro-Kellie hypothesis
B) Glasgow Coma Scale
C) Cranial nerve function
D) Mental status examination
Ans: B
LOC, a sensitive indicator of neurologic function, is assessed based on the criteria in the Glasgow Coma Scale: eye opening, verbal response,
and motor response. The Monro-Kellie hypothesis states that because of the limited space for expansion within the skull, an increase in any
one of the components (blood, brain tissue, cerebrospinal fluid) causes a change in the volume of the others. Cranial nerve function and the
mental status examination would be part of the neurologic examination for this patient, but would not be the priority in evaluating LOC.

10. While completing a health history on a patient who has recently experienced a seizure, the nurse would assess for what characteristic
associated with the postictal state?
A) Epileptic cry
B) Confusion
C) Urinary incontinence
D) Body rigidity
Ans: B
In the postictal state (after the seizure), the patient is often confused and hard to arouse and may sleep for hours. The epileptic cry occurs from
the simultaneous contractions of the diaphragm and chest muscles that occur during the seizure. Urinary incontinence and intense rigidity of
the entire body are followed by alternating muscle relaxation and contraction (generalized tonicclonic contraction) during the seizure.

11. A patient with increased ICP has a ventriculostomy for monitoring ICP. The nurses most recent assessment reveals that the patient is now
exhibiting nuchal rigidity and photophobia. The nurse would be correct in suspecting the presence of what complication?
A) Encephalitis
B) CSF leak
C) Meningitis
D) Catheter occlusion
Ans: C
Complications of a ventriculostomy include ventricular infectious meningitis and problems with the monitoring system. Nuchal rigidity and
photophobia are clinical manifestations of meningitis, but are not suggestive of encephalitis, a CSF leak, or an occluded catheter.

12. The nurse is participating in the care of a patient with increased ICP. What diagnostic test is contraindicated in this patients treatment?
A) Computed tomography (CT) scan
B) Lumbar puncture
C) Magnetic resonance imaging (MRI)
D) Venous Doppler studies
Ans: B
A lumbar puncture in a patient with increased ICP may cause the brain to herniate from the withdrawal of fluid and change in pressure during
the lumbar puncture. Herniation of the brain is a dire and frequently fatal event. CT, MRI, and venous Doppler are considered noninvasive
procedures and they would not affect the ICP itself.

13. The nurse is caring for a pt who is in status epilepticus. What med does the nurse know may be given to halt the seizure immediately?
A) Intravenous phenobarbital (Luminal)
B) Intravenous diazepam (Valium)
C) Oral lorazepam (Ativan)
D) Oral phenytoin (Dilantin)
Ans: B
Medical management of status epilepticus includes IV diazepam (Valium) and IV lorazepam (Ativan) given slowly in an attempt to halt
seizures immediately. Other medications (phenytoin, phenobarbital) are given later to maintain a seizure-free state. Oral medications are not
given during status epilepticus.

14. The nurse has created a plan of care for a patient who is at risk for increased ICP. The patients care plan should specify monitoring for
what early sign of increased ICP?
A) Disorientation and restlessness
B) Decreased pulse and respirations
C) Projectile vomiting
D) Loss of corneal reflex
Ans: A
Early indicators of ICP include disorientation and restlessness. Later signs include decreased pulse and respirations, projectile vomiting, and
loss of brain stem reflexes, such as the corneal reflex.

15. The neurologic ICU nurse is admitting a patient following a craniotomy using the supratentorial approach. How should the nurse best
position the patient?
A) Position the patient supine.
B) Maintain head of bed (HOB) elevated at 30 to 45 degrees.
C) Position patient in prone position.
D) Maintain bed in Trendelenberg position.
Ans: B
The patient undergoing a craniotomy with a supratentorial (above the tentorium) approach should be placed with the HOB elevated 30 to 45
degrees, with the neck in neutral alignment. Each of the other listed positions would cause a dangerous elevation in ICP.

16. A clinic nurse is caring for a patient diagnosed with migraine headaches. During the patient teaching session, the patient questions the
nurse regarding alcohol consumption. What would the nurse be correct in telling the patient about the effects of alcohol?
A) Alcohol causes hormone fluctuations.
B) Alcohol causes vasodilation of the blood vessels.
C) Alcohol has an excitatory effect on the CNS.
D) Alcohol diminishes endorphins in the brain.
Ans: B
Alcohol causes vasodilation of the blood vessels and may exacerbate migraine headaches. Alcohol has a depressant effect on the CNS.
Alcohol does not cause hormone fluctuations, nor does it decrease endorphins (morphine-like substances produced by the body) in the brain.

17. A pt has developed diabetes insipidus after having increased ICP following head trauma. What nursing assessment best addresses this
complication?
A) Vigilant monitoring of fluid balance
B) Continuous BP monitoring
C) Serial arterial blood gases (ABGs)
D) Monitoring of the patients airway for patency
Ans: A
Diabetes insipidus requires fluid and electrolyte replacement, along with the administration of vasopressin, to replace and slow the urine
output. Because of these alterations in fluid balance, careful monitoring is necessary. None of the other listed assessments directly addresses
the major manifestations of diabetes insipidus.

18. What should the nurse suspect when hourly assessment of urine output on a patient postcraniotomy exhibits a urine output from a catheter
of 1,500 mL for two consecutive hours?
A) Cushing syndrome
B) Syndrome of inappropriate antidiuretic hormone (SIADH)
C) Adrenal crisis
D) Diabetes insipidus
Ans: D
Diabetes insipidus is an abrupt onset of extreme polyuria that commonly occurs in patients after brain surgery. Cushing syndrome is excessive
glucocorticoid secretion resulting in sodium and water retention. SIADH is the result of increased secretion of ADH; the patient becomes
volume-overloaded, urine output diminishes, and serum sodium concentration becomes dilute. Adrenal crisis is undersecretion of
glucocorticoids resulting in profound hypoglycemia, hypovolemia, and hypotension.
19. During the examination of an unconscious patient, the nurse observes that the patients pupils are fixed and dilated. What is the most
plausible clinical significance of the nurses finding?
A) It suggests onset of metabolic problems.
B) It indicates paralysis on the right side of the body.
C) It indicates paralysis of cranial nerve X.
D) It indicates an injury at the midbrain level.
Ans: D
Pupils that are fixed and dilated indicate injury at the midbrain level. This finding is not suggestive of unilateral paralysis, metabolic deficits,
or damage to CN X.

20. Following traumatic brain injury, a pt has been in a coma for several days. Which of the following statements is true of this pts current
LOC?
A) The patient occasionally makes incomprehensible sounds.
B) The patients current LOC will likely become a permanent state.
C) The patient may occasionally make nonpurposeful movements.
D) The patient is incapable of spontaneous respirations.
Ans: C
Coma is a clinical state of unarousable unresponsiveness in which no purposeful responses to internal or external stimuli occur, although
nonpurposeful responses to painful stimuli and brain stem reflexes may be present. Verbal sounds, however, are atypical. Ventilator support
may or may not be necessary. Comas are not permanent states.

21. The nurse is caring for a patient with permanent neurologic impairments resulting from a traumatic head injury. When working with this
patient and family, what mutual goal should be prioritized?
A) Achieve as high a level of function as possible.
B) Enhance the quantity of the patients life.
C) Teach the family proper care of the patient.
D) Provide community assistance.
Ans: A
The overarching goals of care are to achieve as high a level of function as possible and to enhance the quality of life for the patient with
neurologic impairment and his or her family. This goal encompasses family and community participation.

22. The nurse is providing care for a pt who is withdrawing from heavy alcohol use. The nurse & other members of the care team are present
at the bedside when the pt has a seizure. In preparation for documenting this clinical event, the nurse should note which of the following?
A) The ability of the patient to follow instructions during the seizure.
B) The success or failure of the care team to physically restrain the patient.
C) The patients ability to explain his seizure during the postictal period.
D) The patients activities immediately prior to the seizure.
Ans: D
Before and during a seizure, the nurse observes the circumstances before the seizure, including visual, auditory, or olfactory stimuli; tactile
stimuli; emotional or psychological disturbances; sleep; and hyperventilation. Communication with the patient is not possible during a seizure
and physical restraint is not attempted. The patients ability to explain the seizure is not clinically relevant.

23. The nurse is caring for a patient whose recent health history includes an altered LOC. What should be
the nurses first action when assessing this patient?
A) Assessing the patients verbal response
B) Assessing the patients ability to follow complex commands
C) Assessing the patients judgment
D) Assessing the patients response to pain
Ans: A
Assessment of the patient with an altered LOC often starts with assessing the verbal response through determining the patients orientation to
time, person, and place. In most cases, this assessment will precede each of the other listed assessments, even though each may be indicated.

24. The nurse caring for a patient in a persistent vegetative state is regularly assessing for potential complications. Complications of
neurologic dysfunction for which the nurse should assess include which of the following? Select all that apply.
A) Contractures
B) Hemorrhage
C) Pressure ulcers
D) Venous thromboembolism
E) Pneumonia
Ans: A, C, D, E
Based on the assessment data, potential complications may include respiratory distress or failure, pneumonia, aspiration, pressure ulcer, deep
vein thrombosis (DVT), and contractures. The pathophysiology of decreased LOC does not normally create a heightened risk for hemorrhage.

25. The nurse is caring for a patient with a brain tumor. What drug would the nurse expect to be ordered to reduce the edema surrounding the
tumor?
A) Solumedrol
B) Dextromethorphan
C) Dexamethasone
D) Furosemide
Ans: C
If a brain tumor is the cause of the increased ICP, corticosteroids (e.g., dexamethasone) help reduce the edema surrounding the tumor.
Solumedrol, a steroid, & furosemide, a loop diuretic, are not the drugs of choice in this instance. Dextromethorphan is used in cough med.

26. The nurse is caring for a patient who sustained a moderate head injury following a bicycle accident. The nurses most recent assessment
reveals that the patients respiratory effort has increased. What is the nurses most appropriate response?
A) Inform the care team and assess for further signs of possible increased ICP.
B) Administer bronchodilators as ordered and monitor the patients LOC.
C) Increase the patients bed height and reassess in 30 minutes.
D) Administer a bolus of normal saline as ordered.
Ans: A
Increased respiratory effort can be suggestive of increasing ICP, and the care team should be promptly informed. A bolus of IV fluid will not
address the problem. Repositioning the patient and administering bronchodilators are insufficient responses, even though these actions may
later be ordered.

27. A patient has a poor prognosis after being involved in a motor vehicle accident resulting in a head injury. As the patients ICP increases
and condition worsens, the nurse knows to assess for indications of approaching death. These indications include which of the following?
A) Hemiplegia
B) Dry mucous membranes
C) Signs of internal bleeding
D) Loss of brain stem reflexes
Ans: D
Loss of brain stem reflexes, including pupillary, corneal, gag, and swallowing reflexes, is an ominous sign of approaching death. Dry mucous
membranes, hemiplegia, and bleeding must be promptly addressed, but none of these is a common sign of impending death.

28. A patient has experienced a seizure in which she became rigid and then experienced alternating muscle relaxation and contraction. What
type of seizure does the nurse recognize?
A) Unclassified seizure
B) Absence seizure
C) Generalized seizure
D) Focal seizure
Ans: C
Generalized seizures often involve both hemispheres of the brain, causing both sides of the body to react. Intense rigidity of the entire body
may occur, followed by alternating muscle relaxation and contraction (generalized tonicclonic contraction). This pattern of rigidity does not
occur in patients who experience unclassified, absence, or focal seizures.

29. When caring for a patient with increased ICP the nurse knows the importance of monitoring for possible secondary complications,
including syndrome of inappropriate antidiuretic hormone (SIADH). What nursing interventions would the nurse most likely initiate if the
patient developed SIADH?
A) Fluid restriction
B) Transfusion of platelets
C) Transfusion of fresh frozen plasma (FFP)
D) Electrolyte restriction
Ans: A
The nurse also assesses for complications of increased ICP, including diabetes insipidus, and SIADH. SIADH requires fluid restriction and
monitoring of serum electrolyte levels. Transfusions are unnecessary.

30. The nurse is admitting a patient to the unit who is scheduled for removal of an intracranial mass. What diagnostic procedures might be
included in this patients admission orders? Select all that apply.
A) Transcranial Doppler flow study
B) Cerebral angiography
C) MRI
D) Cranial radiography
E) Electromyelography (EMG)
Ans: A, B, C
Preoperative diagnostic procedures may include a CT scan to demonstrate the lesion and show the degree of surrounding brain edema, the
ventricular size, and the displacement. An MRI scan provides information similar to that of a CT scan with improved tissue contrast,
resolution, and anatomic definition. Cerebral angiography may be used to study a tumors blood supply or to obtain information about
vascular lesions. Transcranial Doppler flow studies are used to evaluate the blood flow within intracranial blood vessels. Regular x-rays of the
skull would not be diagnostic for an intracranial mass. An EMG would not be ordered prior to intracranial surgery to remove a mass.
31. A patient is recovering from intracranial surgery performed approximately 24 hours ago and is complaining of a headache that the patient
rates at 8 on a 10-point pain scale. What nursing action is most appropriate?
A) Administer morphine sulfate as ordered.
B) Reposition the patient in a prone position.
C) Apply a hot pack to the patients scalp.
D) Implement distraction techniques.
Ans: A
The patient usually has a headache after a craniotomy as a result of stretching and irritation of nerves in the scalp during surgery. Morphine
sulfate may also be used in the management of postoperative pain in patients who have undergone a craniotomy. Prone positioning is
contraindicated due to the consequent increase in ICP. Distraction would likely be inadequate to reduce pain and a hot pack may cause
vasodilation and increased pain.

32. A patient is recovering from intracranial surgery that was performed using the transsphenoidal approach. The nurse should be aware that
the patient may have required surgery on what neurologic structure?
A) Cerebellum
B) Hypothalamus
C) Pituitary gland
D) Pineal gland
Ans: C
The transsphenoidal approach (through the mouth and nasal sinuses) is often used to gain access to the pituitary gland. This surgical approach
would not allow for access to the pineal gland, cerebellum, or hypothalamus.

33. A patient is postoperative day 1 following intracranial surgery. The nurses assessment reveals that the patients LOC is slightly decreased
compared with the day of surgery. What is the nurses best response to this assessment finding?
A) Recognize that this may represent the peak of post-surgical cerebral edema.
B) Alert the surgeon to the possibility of an intracranial hemorrhage.
C) Understand that the surgery may have been unsuccessful.
D) Recognize the need to refer the patient to the palliative care team.
Ans: A
Some degree of cerebral edema occurs after brain surgery; it tends to peak 24 to 36 hours after surgery, producing decreased responsiveness
on the second postoperative day. As such, there is not necessarily any need to deem the surgery unsuccessful or to refer the patient to
palliative care. A decrease in LOC is not evidence of an intracranial hemorrhage.

34. A school nurse is called to the playground where a 6-year-old girl has been found unresponsive and staring into space, according to the
playground supervisor. How would the nurse document the girls activity in her chart at school?
A) Generalized seizure
B) Absence seizure
C) Focal seizure
D) Unclassified seizure
Ans: B
Staring episodes characterize an absence seizure, whereas focal seizures, generalized seizures, and unclassified seizures involve uncontrolled
motor activity.

35. A neurologic nurse is reviewing seizures with a group of staff nurses. How should this nurse best describe the cause of a seizure?
A) Sudden electrolyte changes throughout the brain
B) A dysrhythmia in the peripheral nervous system
C) A dysrhythmia in the nerve cells in one section of the brain
D) Sudden disruptions in the blood flow throughout the brain
Ans: C
The underlying cause of a seizure is an electrical disturbance (dysrhythmia) in the nerve cells in one section of the brain; these cells emit
abnormal, recurring, uncontrolled electrical discharges. Seizures are not caused by changes in blood flow or electrolytes.

36. The nurse is caring for a patient who has undergone supratentorial removal of a pituitary mass. What medication would the nurse expect
to administer prophylactically to prevent seizures in this patient?
A) Prednisone
B) Dexamethasone
C) Cafergot
D) Phenytoin
Ans: D
Antiseizure medication (phenytoin, diazepam) is often prescribed prophylactically for patients who have undergone supratentorial craniotomy
because of the high risk of seizures after this procedure. Prednisone and dexamethasone are steroids and do not prevent seizures. Cafergot is
used in the treatment of migraines.
37. A hospital patient has experienced a seizure. In the immediate recovery period, what action best protects the patients safety?
A) Place the patient in a side-lying position.
B) Pad the patients bed rails.
C) Administer antianxiety medications as ordered.
D) Reassure the patient and family members.
Ans: A
To prevent complications, the pt placed in the side-lying position to facilitate drainage of oral secretions. Suctioning is performed, if needed,
to maintain a patent airway & prevent aspiration. None of the other listed actions promotes safety during the immediate recovery period.

38. A nurse is caring for a patient who experiences debilitating cluster headaches. The patient should be taught to take appropriate
medications at what point in the course of the onset of a new headache?
A) As soon as the patients pain becomes unbearable
B) As soon as the patient senses the onset of symptoms
C) Twenty to 30 minutes after the onset of symptoms
D) When the patient senses his or her symptoms peaking
Ans: B
A migraine or a cluster headache in the early phase requires abortive medication therapy instituted as soon as possible. Delaying medication
administration would lead to unnecessary pain.

39. A nurse is collaborating with the interdisciplinary team to help manage a patients recurrent headaches. What aspect of the patients health
history should the nurse identify as a potential contributor to the patients headaches?
A) The patient leads a sedentary lifestyle.
B) The patient takes vitamin D and calcium supplements.
C) The patient takes vasodilators for the treatment of angina.
D) The patient has a pattern of weight loss followed by weight gain.
Ans: C
Vasodilators are known to contribute to HA. Weight fluctuations, sedentary lifestyle, & vit supplements are not known to have this effect.

40. An adult patient has sought care for the treatment of headaches that have become increasingly severe and frequent over the past several
months. Which of the following questions addresses potential etiological factors? Select all that apply?
A) Are you exposed to any toxins or chemicals at work?
B) How would you describe your ability to cope with stress?
C) What medications are you currently taking?
D) When was the last time you were hospitalized?
E) Does anyone else in your family struggle with headaches?
Ans: A, B, C, E
Headaches are multifactorial, and may involve medications, exposure to toxins, family history, and stress. Hospitalization is an unlikely
contributor to headaches.
Chapter 67: Management of Patients with Cerebrovascular Disorders
1. A pt has had ischemic stroke & has been admitted to medical unit. What action should the nurse perform to best prevent joint deformities?
A) Place the patient in the prone position for 30 minutes/day.
B) Assist the patient in acutely flexing the thigh to promote movement.
C) Place a pillow in the axilla when there is limited external rotation.
D) Place patients hand in pronation.
Ans: C
A pillow in the axilla prevents adduction of the affected shoulder and keeps the arm away from the chest. The prone position with a pillow
under the pelvis, not flat, promotes hyperextension of the hip joints, essential for normal gait. To promote venous return and prevent edema,
the upper thigh should not be flexed acutely. The hand is placed in slight supination, not pronation, which is its most functional position.

2. A patient diagnosed with transient ischemic attacks (TIAs) is scheduled for a carotid endarterectomy. The nurse explains that this procedure
will be done for what purpose?
A) To decrease cerebral edema
B) To prevent seizure activity that is common following a TIA
C) To remove atherosclerotic plaques blocking cerebral flow
D) To determine the cause of the TIA
Ans: C
The main surgical procedure for select patients with TIAs is carotid endarterectomy, the removal of an atherosclerotic plaque or thrombus
from the carotid artery to prevent stroke in patients with occlusive disease of the extracranial arteries. An endarterectomy does not decrease
cerebral edema, prevent seizure activity, or determine the cause of a TIA.

3. The nurse is discharging home a patient who suffered a stroke. He has a flaccid right arm and leg & is experiencing problems w/urinary
incontinence. The nurse makes a referral to a home health nurse b/c of an awareness of what common pt response to a change in body image?
A) Denial
B) Fear
C) Depression
D) Disassociation
Ans: C
Depression is a common and serious problem in the patient who has had a stroke. It can result from a profound disruption in his or her life
and changes in total function, leaving the patient with a loss of independence. The nurse needs to encourage the patient to verbalize feelings
to assess the effect of the stroke on self-esteem. Denial, fear, and disassociation are not the most common patient response to a change in
body image, although each can occur in some patients.

4. When caring for a patient who had a hemorrhagic stroke, close monitoring of vital signs and neurologic changes is imperative. What is the
earliest sign of deterioration in a patient with a hemorrhagic stroke of which the nurse should be aware?
A) Generalized pain
B) Alteration in level of consciousness (LOC)
C) Tonicclonic seizures
D) Shortness of breath
Ans: B
Alteration in LOC is the earliest sign of deterioration in a patient after a hemorrhagic stroke, such as mild drowsiness, slight slurring of
speech, and sluggish papillary reaction. Sudden headache may occur, but generalized pain is less common. Seizures and shortness of breath
are not identified as early signs of hemorrhagic stroke.

5. The nurse is performing stroke risk screenings at a hospital open house. The nurse has identified four patients who might be at risk for a
stroke. Which patient is likely at the highest risk for a hemorrhagic stroke?
A) White female, age 60, with history of excessive alcohol intake
B) White male, age 60, with history of uncontrolled hypertension
C) Black male, age 60, with history of diabetes
D) Black male, age 50, with history of smoking
Ans: B
Uncontrolled hypertension is the primary cause of a hemorrhagic stroke. Control of hypertension, especially in individuals over 55 years of
age, clearly reduces the risk for hemorrhagic stroke. Additional risk factors are increased age, male gender, and excessive alcohol intake.
Another high-risk group includes African Americans, where the incidence of first stroke is almost twice that as in Caucasians.

6. A pt who just suffered suspected ischemic stroke is brought to the ED by ambulance. On what should the nurses primary assessment focus?
A) Cardiac and respiratory status
B) Seizure activity
C) Pain
D) Fluid and electrolyte balance
Ans: A
Acute care begins with managing ABCs. Patients may have difficulty keeping an open and clear airway secondary to decreased LOC.
Neurologic assessment with close monitoring for signs of increased neurologic deficit and seizure activity occurs next. Fluid and electrolyte
balance must be controlled carefully with the goal of adequate hydration to promote perfusion and decrease further brain activity.

7. A patient with a cerebral aneurysm exhibits signs and symptoms of an increase in intracranial pressure (ICP). What nursing intervention
would be most appropriate for this patient?
A) Range-of-motion exercises to prevent contractures
B) Encouraging independence with ADLs to promote recovery
C) Early initiation of physical therapy
D) Absolute bed rest in a quiet, nonstimulating environment
Ans: D
The patient is placed on immediate and absolute bed rest in a quiet, nonstressful environment because activity, pain, and anxiety elevate BP,
which increases the risk for bleeding. Visitors are restricted. The nurse administers all personal care. The patient is fed and bathed to prevent
any exertion that might raise BP.

8. A patient recovering from a stroke has severe shoulder pain from subluxation of the shoulder and is being cared for on the unit. To prevent
further injury and pain, the nurse caring for this patient is aware of what principle of care?
A) The patient should be fitted with a cast because use of a sling should be avoided due to adduction of the affected shoulder.
B) Elevation of the arm and hand can lead to further complications associated with edema.
C) Passively exercising the affected extremity is avoided in order to minimize pain.
D) The patient should be taught to interlace fingers, place palms together, and slowly bring scapulae
forward to avoid excessive force to shoulder.
Ans: D
To prevent shoulder pain, the nurse should never lift a patient by the flaccid shoulder or pull on the affected arm or shoulder. The patient is
taught how to move and exercise the affected arm/shoulder through proper movement and positioning. The patient is instructed to interlace
the fingers, place the palms together, and push the clasped hands slowly forward to bring the scapulae forward; he or she then raises both
hands above the head. This is repeated throughout the day. The use of a properly worn sling when the patient is out of bed prevents the
paralyzed upper extremity from dangling without support. Range-of-motion exercises are still vitally important in preventing a frozen
shoulder and ultimately atrophy of subcutaneous tissues, which can cause more pain. Elevation of the arm and hand is also important in
preventing dependent edema of the hand.

9. The patient has been diagnosed with aphasia after suffering a stroke. What can the nurse do to best make the patients atmosphere more
conducive to communication?
A) Provide a board of commonly used needs and phrases.
B) Have the patient speak to loved ones on the phone daily.
C) Help the patient complete his or her sentences.
D) Speak in a loud and deliberate voice to the patient.
Ans: A
The inability to talk on the telephone or answer a question or exclusion from conversation causes anger, frustration, fear of the future, and
hopelessness. A common pitfall is for the nurse or other health care team member to complete the thoughts or sentences of the patient. This
should be avoided because it may cause the patient to feel more frustrated at not being allowed to speak and may deter efforts to practice
putting thoughts together and completing a sentence. The patient may also benefit from a communication board, which has pictures of
commonly requested needs and phrases. The board may be translated into several languages.

10. The nurse is assessing a patient with a suspected stroke. What assessment finding is most suggestive of a stroke?
A) Facial droop
B) Dysrhythmias
C) Periorbital edema
D) Projectile vomiting
Ans: A
Facial drooping or asymmetry is a classic abnormal finding on a physical assessment that may be associated with a stroke. Facial edema is
not suggestive of a stroke and patients less commonly experience dysrhythmias or vomiting.

11. The nurse is caring for a patient diagnosed with an ischemic stroke and knows that effective positioning of the patient is important. Which
of the following should be integrated into the patients plan of care?
A) The patients hip joint should be maintained in a flexed position.
B) The patient should be in a supine position unless ambulating.
C) The patient should be placed in a prone position for 15 to 30 minutes several times a day.
D) The patient should be placed in a Trendelenberg position two to three times daily to promote cerebral perfusion.
Ans: C
If possible, the patient is placed in a prone position for 15 to 30 minutes several times a day. A small pillow or a support is placed under the
pelvis, extending from the level of the umbilicus to the upper third of the thigh. This helps to promote hyperextension of the hip joints, which
is essential for normal gait, and helps prevent knee and hip flexion contractures. The hip joints should not be maintained in flexion and the
Trendelenberg position is not indicated.
12. A patient has been admitted to the ICU after being recently diagnosed with an aneurysm and the patients admission orders include specific
aneurysm precautions. What nursing action will the nurse incorporate into the patients plan of care?
A) Elevate the head of the bed to 45 degrees.
B) Maintain the patient on complete bed rest.
C) Administer enemas when the patient is constipated.
D) Avoid use of thigh-high elastic compression stockings.
Ans: B
Cerebral aneurysm precautions are implemented for the patient with a diagnosis of aneurysm to provide a nonstimulating environment,
prevent increases in ICP, and prevent further bleeding. The patient is placed on immediate and absolute bed rest in a quiet, nonstressful
environment because activity, pain, and anxiety elevate BP, which increases the risk for bleeding. Visitors, except for family, are restricted.
The head of the bed is elevated 15 to 30 degrees to promote venous drainage and decrease ICP. Some neurologists, however, prefer that the
patient remains flat to increase cerebral perfusion. No enemas are permitted, but stool softeners and mild laxatives are prescribed. Thigh-high
elastic compression stockings or sequential compression boots may be ordered to decrease the patients risk for deep vein thrombosis (DVT).

13. A nurse is caring for a pt diagnosed with a hemorrhagic stroke. When creating this patients plan of care, what goal should be prioritized?
A) Prevent complications of immobility.
B) Maintain and improve cerebral tissue perfusion.
C) Relieve anxiety and pain.
D) Relieve sensory deprivation.
Ans: B
Each of the listed goals is appropriate in the care of a patient recovering from a stroke. However, promoting cerebral perfusion is a priority
physiologic need, on which the patients survival depends.

14. The nurse is preparing health education for a patient who is being discharged after hospitalization for a hemorrhagic stroke. What content
should the nurse include in this education?
A) Mild, intermittent seizures can be expected.
B) Take ibuprofen for complaints of a serious headache.
C) Take antihypertensive medication as ordered.
D) Drowsiness is normal for the first week after discharge.
Ans: C
The patient and family are provided with information that will enable them to cooperate with the care and restrictions required during the
acute phase of hemorrhagic stroke and to prepare the patient to return home. Patient and family teaching includes information about the
causes of hemorrhagic stroke and its possible consequences. Symptoms of hydrocephalus include gradual onset of drowsiness and behavioral
changes. Hypertension is the most serious risk factor, suggesting that appropriate antihypertensive treatment is essential for a patient being
discharged. Seizure activity is not normal; complaints of a serious headache should be reported to the physician before any medication is
taken. Drowsiness is not normal or expected.

15. A patient diagnosed with a cerebral aneurysm reports a severe headache to the nurse. What action is a priority for the nurse?
A) Sit with the patient for a few minutes.
B) Administer an analgesic.
C) Inform the nurse-manager.
D) Call the physician immediately.
Ans: D
A headache may be an indication that the aneurysm is leaking. The nurse should notify the physician immediately. The physician will decide
whether administration of an analgesic is indicated. Informing the nurse-manager is not necessary. Sitting with the patient is appropriate, once
the physician has been notified of the change in the patients condition.

16. A patient is brought by ambulance to the ED after suffering what the family thinks is a stroke. The nurse caring for this patient is aware
that an absolute contraindication for thrombolytic therapy is what?
A) Evidence of hemorrhagic stroke
B) Blood pressure of 180/110 mm Hg
C) Evidence of stroke evolution
D) Previous thrombolytic therapy within the past 12 months
Ans: A
Thrombolytic therapy would exacerbate a hemorrhagic stroke with potentially fatal consequences. Stroke evolution, high BP, or previous
thrombolytic therapy does not contraindicate its safe and effective use.

17. When caring for a patient who has had a stroke, a priority is reduction of ICP. What patient position is most consistent with this goal?
A) Head turned slightly to the right side
B) Elevation of the head of the bed
C) Position changes every 15 minutes while awake
D) Extension of the neck
Ans: B
Elevation of the head of the bed promotes venous drainage and lowers ICP; the nurse should avoid flexing or extending the neck or turning
the head side to side. The head should be in a neutral midline position. Excessively frequent position changes are unnecessary.
18. A pt who suffered an ischemic stroke now has disturbed sensory perception. What principle should guide the nurses care of this patient?
A) The patient should be approached on the side where visual perception is intact.
B) Attention to the affected side should be minimized in order to decrease anxiety.
C) The patient should avoid turning in the direction of the defective visual field to minimize shoulder subluxation.
D) The patient should be approached on the opposite side of where the visual perception is intact to promote recovery.
Ans: A
Patients with decreased field of vision should first be approached on the side where visual perception is intact. All visual stimuli should be
placed on this side. The patient can and should be taught to turn the head in the direction of the defective visual field to compensate for this
loss. The nurse should constantly remind the patient of the other side of the body and should later stand at a position that encourages the
patient to move or turn to visualize who and what is in the room.

19. What should be included in the patients care plan when establishing an exercise program for a patient affected by a stroke?
A) Schedule passive range of motion every other day.
B) Keep activity limited, as the patient may be over stimulated.
C) Have the patient perform active range-of-motion (ROM) exercises once a day.
D) Exercise the affected extremities passively four or five times a day.
Ans: D
The affected extremities are exercised passively and put through a full ROM four or five times a day to maintain joint mobility, regain motor
control, prevent development of a contracture in the paralyzed extremity, prevent further deterioration of the neuromuscular system, and
enhance circulation. Active ROM exercises should ideally be performed more than once per day.

20. A female patient is diagnosed with a right-sided stroke. The patient is now experiencing hemianopsia. How might the nurse help the
patient manage her potential sensory and perceptional difficulties?
A) Keep the lighting in the patients room low.
B) Place the patients clock on the affected side.
C) Approach the patient on the side where vision is impaired.
D) Place the patients extremities where she can see them.
Ans: D
The patient with homonymous hemianopsia (loss of half of the visual field) turns away from the affected side of the body and tends to neglect
that side and the space on that side; this is called amorphosynthesis. In such instances, the patient cannot see food on half of the tray, and only
half of the room is visible. It is important for the nurse to remind the patient constantly of the other side of the body, to maintain alignment of
the extremities, and if possible, to place the extremities where the patient can see them. Patients with a decreased field of vision should be
approached on the side where visual perception is intact. All visual stimuli (clock, calendar, and television) should be placed on this side. The
patient can be taught to turn the head in the direction of the defective visual field to compensate for this loss. Increasing the natural or
artificial lighting in the room and providing eyeglasses are important in increasing vision. There is no reason to keep the lights dim.

21. The public health nurse is planning a health promotion campaign that reflects current epidemiologic trends. The nurse should know that
hemorrhagic stroke currently accounts for what percentage of total strokes in the United States?
A) 43%
B) 33%
C) 23%
D) 13%
Ans: D
Strokes can be divided into two major categories: ischemic (87%), in which vascular occlusion and significant hypoperfusion occur, and
hemorrhagic (13%), in which there is extravasation of blood into the brain or subarachnoid space.

22. A patient who has experienced an ischemic stroke has been admitted to the medical unit. The patients family in adamant that she remain
on bed rest to hasten her recovery and to conserve energy. What principle of care should inform the nurses response to the family?
A) The patient should mobilize as soon as she is physically able.
B) To prevent contractures and muscle atrophy, bed rest should not exceed 4 weeks.
C) The patient should remain on bed rest until she expresses a desire to mobilize.
D) Lack of mobility will greatly increase the patients risk of stroke recurrence.
Ans: A
As soon as possible, the patient is assisted out of bed and an active rehabilitation program is started. Delaying mobility causes complications,
but not necessarily stroke recurrence. Mobility should not be withheld until the patient initiates.

23. A patient has recently begun mobilizing during the recovery from an ischemic stroke. To protect the patients safety during mobilization,
the nurse should perform what action?
A) Support the patients full body weight with a waist belt during ambulation.
B) Have a colleague follow the patient closely with a wheelchair.
C) Avoid mobilizing the patient in the early morning or late evening.
D) Ensure that the patients family members do not participate in mobilization.
Ans: B
During mobilization, a chair or wheelchair should be readily available in case the patient suddenly becomes fatigued or feels dizzy. The
family should be encouraged to participate, as appropriate, and the nurse should not have to support the patients full body weight. Morning
and evening activity are not necessarily problematic.
24. A pt diagnosed with a hemorrhagic stroke has been admitted to neurologic ICU. The nurse knows that teaching for the pt & family needs
to begin as soon as the pt is settled on the unit and will continue until the patient is discharged. What will family education need to include?
A) How to differentiate between hemorrhagic and ischemic stroke
B) Risk factors for ischemic stroke
C) How to correctly modify the home environment
D) Techniques for adjusting the patients medication dosages at home
Ans: C
For a patient with a hemorrhagic stroke, teaching addresses the use of assistive devices or modification of the home environment to help the
patient live with the disability. This is more important to the patients needs than knowing about risk factors for ischemic stroke. It is not
necessary for the family to differentiate between different types of strokes. Medication regimens should never be altered without consultation.

25. After a subarachnoid hemorrhage, the patients laboratory results indicate a serum sodium level of less than 126 mEq/L. What is the nurses
most appropriate action?
A) Administer a bolus of normal saline as ordered.
B) Prepare the patient for thrombolytic therapy as ordered.
C) Facilitate testing for hypothalamic dysfunction.
D) Prepare to administer 3% NaCl by IV as ordered.
Ans: D
The patient may be experiencing syndrome of inappropriate antidiuretic hormone (SIADH) or cerebral salt-wasting syndrome. The treatment
most often is the use of IV hypertonic 3% saline. A normal saline bolus would exacerbate the problem and there is no indication for tests of
hypothalamic function or thrombolytic therapy.

26. A community health nurse is giving an educational presentation about stroke and heart disease at the local senior citizens center. What
nonmodifiable risk factor for stroke should the nurse cite?
A) Female gender
B) Asian American race
C) Advanced age
D) Smoking
Ans: C
Advanced age, male gender, and race are well-known nonmodifiable risk factors for stroke. High-risk groups include people older than 55
years of age; the incidence of stroke more than doubles in each successive decade. Men have a higher rate of stroke than that of women.
Another high-risk group is African Americans; the incidence of first stroke in African Americans is almost twice that as in Caucasian
Americans; Asian American race is not a risk factor. Smoking is a modifiable risk.

27. A family member brings the patient to the clinic for a follow-up visit after a stroke. The family member asks the nurse what he can do to
decrease his chance of having another stroke. What would be the nurses best answer?
A) Have your heart checked regularly.
B) Stop smoking as soon as possible.
C) Get medication to bring down your sodium levels.
D) Eat a nutritious diet.
Ans: B
Smoking is a modifiable and highly significant risk factor for stroke. The significance of smoking, and the potential benefits of quitting,
exceed the roles of sodium, diet, and regular medical assessments.

28. The nurse is reviewing the medication administration record of a female patient who possesses numerous risk factors for stroke. Which of
the womans medications carries the greatest potential for reducing her risk of stroke?
A) Naproxen 250 PO b.i.d.
B) Calcium carbonate 1,000 mg PO b.i.d.
C) Aspirin 81 mg PO o.d.
D) Lorazepam 1 mg SL b.i.d. PRN
Ans: C
Research findings suggest that low-dose aspirin may lower the risk of stroke in women who are at risk. Naproxen, lorazepam, and calcium
supplements do not have this effect.

29. A nurse in the ICU is providing care for a patient who has been admitted with a hemorrhagic stroke. The nurse is performing frequent
neurologic assessments and observes that the patient is becoming progressively more drowsy over the course of the day. What is the nurses
best response to this assessment finding?
A) Report this finding to the physician as an indication of decreased metabolism.
B) Provide more stimulation to the patient and monitor the patient closely.
C) Recognize this as the expected clinical course of a hemorrhagic stroke.
D) Report this to the physician as a possible sign of clinical deterioration.
Ans: D
Alteration in LOC often is the earliest sign of deterioration in a patient with a hemorrhagic stroke. Drowsiness and slight slurring of speech
may be early signs that the LOC is deteriorating. This finding is unlikely to be the result of metabolic changes and it is not expected.
Stimulating a patient with an acute stroke is usually contraindicated.
30. Following diagnostic testing, a patient has been admitted to the ICU and placed on cerebral aneurysm precautions. What nursing action
should be included in patients plan of care?
A) Supervise the patients activities of daily living closely.
B) Initiate early ambulation to prevent complications of immobility.
C) Provide a high-calorie, low-protein diet.
D) Perform all of the patients hygiene and feeding.
Ans: A
The patient is placed on immediate and absolute bed rest in a quiet, nonstressful environment, because activity, pain, and anxiety elevate BP,
which increases the risk for bleeding. As such, independent ADLs and ambulation are contraindicated. There is no need for a high-calorie or
low-protein diet.

31. A preceptor is discussing stroke with a new nurse on the unit. The preceptor would tell the new nurse which cardiac dysrhythmia is
associated with cardiogenic embolic strokes?
A) Ventricular tachycardia
B) Atrial fibrillation
C) Supraventricular tachycardia
D) Bundle branch block
Ans: B
Cardiogenic embolic strokes are associated with cardiac dysrhythmias, usually atrial fibrillation. The other listed dysrhythmias are less
commonly associated with this type of stroke.

32. The pathophysiology of an ischemic stroke involves the ischemic cascade, which includes the following steps:
1. Change in pH
2. Blood flow decreases
3. A switch to anaerobic respiration
4. Membrane pumps fail
5. Cells cease to function
6. Lactic acid is generated
Put these steps in order in which they occur.
A) 635241
B) 352416
C) 236145
D) 162534
Ans: C
The ischemic cascade begins when cerebral blood flow decreases to less than 25 mL per 100 g of blood per minute. At this point, neurons are
no longer able to maintain aerobic respiration. The mitochondria must then switch to anaerobic respiration, which generates large amounts of
lactic acid, causing a change in the pH. This switch to the less efficient anaerobic respiration also renders the neuron incapable of producing
sufficient quantities of adenosine triphosphate (ATP) to fuel the depolarization processes. The membrane pumps that maintain electrolyte
balances begin to fail, and the cells cease to function.

33. As a member of the stroke team, the nurse knows that thrombolytic therapy carries the potential for benefit and for harm. The nurse
should be cognizant of what contraindications for thrombolytic therapy? Select all that apply.
A) INR above 1.0
B) Recent intracranial pathology
C) Sudden symptom onset
D) Current anticoagulation therapy
E) Symptom onset greater than 3 hours prior to admission
Ans: B, D, E
Some of the absolute contraindications for thrombolytic therapy include sx onset greater than 3 hrs before admission, a pt who is
anticoagulated (with INR above 1.7), or pt who has had any type of intracranial pathology (e.g., previous stroke, head injury, trauma).

34. After a major ischemic stroke, a possible complication is cerebral edema. Nursing care during the immediate recovery period from an
ischemic stroke should include which of the following?
A) Positioning to avoid hypoxia
B) Maximizing PaCO2
C) Administering hypertonic IV solution
D) Initiating early mobilization
Ans: A
Interventions during this period include measures to reduce ICP, such as administering an osmotic diuretic (e.g., mannitol), maintaining the
partial pressure of carbon dioxide (PaCO2) within the range of 30 to 35 mm Hg, and positioning to avoid hypoxia. Hypertonic IV solutions
are not used unless sodium depletion is evident. Mobilization would take place after the immediate threat of increased ICP has past.

35. The nurse is caring for a pt recovering from an ischemic stroke. What intervention best addresses a potential complication after an
ischemic stroke?
A) Providing frequent small meals rather than three larger meals
B) Teaching the patient to perform deep breathing and coughing exercises
C) Keeping a urinary catheter in situ for the full duration of recovery
D) Limiting intake of insoluble fiber
Ans: B
Because pneumonia is a potential complication of stroke, deep breathing and coughing exercises should be encouraged unless
contraindicated. No particular need exists to provide frequent meals and normally fiber intake should not be restricted. Urinary catheters
should be discontinued as soon as possible.

36. During a patients recovery from stroke, the nurse should be aware of predictors of stroke outcome in order to help patients and families
set realistic goals. What are the predictors of stroke outcome? Select all that apply.
A) National Institutes of Health Stroke Scale (NIHSS) score
B) Race
C) LOC at time of admission
D) Gender
E) Age
Ans: A, C, E
It is helpful for clinicians to be knowledgeable about the relative importance of predictors of stroke outcome (age, NIHSS score, and LOC at
time of admission) to provide stroke survivors and their families with realistic goals. Race and gender are not predictors of stroke outcome.

37. A nursing student is writing a care plan for a newly admitted patient who has been diagnosed with a stroke. What major nursing diagnosis
should most likely be included in the patients plan of care?
A) Adult failure to thrive
B) Post-trauma syndrome
C) Hyperthermia
D) Disturbed sensory perception
Ans: D
The patient who has experienced a stroke is at a high risk for disturbed sensory perception. Stroke is associated with multiple other nursing
diagnoses, but hyperthermia, adult failure to thrive, and post- trauma syndrome are not among these.

38. When preparing to discharge a patient home, the nurse has met with the family and warned them that the patient may exhibit unexpected
emotional responses. The nurse should teach the family that these responses are typically a result of what cause?
A) Frustration around changes in function and communication
B) Unmet physiologic needs
C) Changes in brain activity during sleep and wakefulness
D) Temporary changes in metabolism
Ans: A
Emotional problems associated with stroke are often related to the new challenges around ADLs and communication. These challenges are
more likely than metabolic changes, unmet physiologic needs, or changes in brain activity, each of which should be ruled out.

39. A rehabilitation nurse caring for a patient who has had a stroke is approached by the patients family and asked why the patient has to do
so much for herself when she is obviously struggling. What would be the nurses best answer?
A) We are trying to help her be as useful as she possibly can.
B) The focus on care in a rehabilitation facility is to help the patient to resume as much self-care as possible.
C) We arent here to care for her the way the hospital staff did; we are here to help her get better so she can go home.
D) Rehabilitation means helping patients do exactly what they did before their stroke.
Ans: B
In both acute care and rehabilitation facilities, the focus is on teaching the patient to resume as much self-care as possible. The goal of
rehabilitation is not to be useful, nor is it to return patients to their prestroke level of functioning, which may be unrealistic.

40. A pt with a new diagnosis of ischemic stroke is deemed to be a candidate for tx w/tissue plasminogen activator (t-PA) & has been
admitted to ICU. In addition to closely monitor the pts cardiac & neurologic status, the nurse monitors the pt for signs of what complication?
A) Acute pain
B) Septicemia
C) Bleeding
D) Seizures
Ans: C
Bleeding is the most common side effect of t-PA administration, and the patient is closely monitored for any bleeding. Septicemia, pain, and
seizures are much less likely to result from thrombolytic therapy.
Chapter 68: Management of Patients with Neurologic Trauma
1. The ED nurse is caring for a patient who has been brought in by ambulance after sustaining a fall at home. What physical assessment
finding is suggestive of a basilar skull fracture?
A) Epistaxis
B) Periorbital edema
C) Bruising over the mastoid
D) Unilateral facial numbness
Ans: C
An area of ecchymosis (bruising) may be seen over the mastoid (Battles sign) in a basilar skull fracture. Numbness, edema, and epistaxis are
not directly associated with a basilar skull fracture.

2. A pt is brought to the trauma center by ambulance after sustaining a high cervical spinal cord injury 1 hours ago. Endotracheal intubation
has been deemed necessary and the nurse is preparing to assist. What nursing diagnosis should the nurse associate with this procedure?
A) Risk for impaired skin integrity
B) Risk for injury
C) Risk for autonomic dysreflexia
D) Risk for suffocation
Ans: B
If endotracheal intubation is necessary, extreme care is taken to avoid flexing or extending the patients neck, which can result in extension of
a cervical injury. Intubation does not directly cause autonomic dysreflexia and the threat to skin integrity is a not a primary concern.
Intubation does not carry the potential to cause suffocation.

3. A nurse is caring for a critically ill patient with autonomic dysreflexia. What clinical manifestations would the nurse expect in this patient?
A) Respiratory distress and projectile vomiting
B) Bradycardia and hypertension
C) Tachycardia and agitation
D) Third-spacing and hyperthermia
Ans: B
Autonomic dysreflexia is characterized by a pounding headache, profuse sweating, nasal congestion, piloerection (goose bumps),
bradycardia, and hypertension. It occurs in cord lesions above T6 after spinal shock has resolved; it does not result in vomiting, tachycardia,
or third-spacing.

4. The nurse is caring for a patient with increased intracranial pressure (ICP) caused by a traumatic brain injury. Which of the following
clinical manifestations would suggest that the patient may be experiencing increased brain compression causing brain stem damage?
A) Hyperthermia
B) Tachycardia
C) Hypertension
D) Bradypnea
Ans: A
Signs of increasing ICP include slowing of the HR (bradycardia), increasing systolic BP, and widening pulse pressure. As brain compression
increases, RR become rapid, BP may decrease, and the pulse slows further. A rapid rise in T is regarded as unfavorable. Hyperthermia
increases the metabolic demands of the brain and may indicate brain stem damage.

5. A patient is brought to the ED by her family after falling off the roof. A family member tells the nurse that when the patient fell she was
knocked out, but came to and seemed okay. Now she is complaining of a severe headache and not feeling well. The care team suspects an
epidural hematoma, prompting the nurse to prepare for which priority intervention?
A) Insertion of an intracranial monitoring device
B) Treatment with antihypertensives
C) Emergency craniotomy
D) Administration of anticoagulant therapy
Ans: C
An epidural hematoma is considered an extreme emergency. Marked neurologic deficit or respiratory arrest can occur within min. Treatment
consists of making an opening through the skull to decrease ICP emergently, remove the clot, and control the bleeding. Antihypertensive
medications would not be a priority. Anticoagulant therapy should not be ordered for a patient who has a cranial bleed. This could further
increase bleeding activity. Insertion of an intracranial monitoring device may be done during the surgery, but is not priority for this patient.

6. The staff educator is precepting a nurse new to the critical care unit when a patient with a T2 spinal cord injury is admitted. The pt is soon
exhibiting manifestations of neurogenic shock. In addition to monitoring the pt closely, what would be the nurses most appropriate action?
A) Prepare to transfuse packed red blood cells.
B) Prepare for interventions to increase the patients BP.
C) Place the patient in the Trendelenberg position.
D) Prepare an ice bath to lower core body temperature.
Ans: B
Manifestations of neurogenic shock include decreased BP and heart rate. Cardiac markers would be expected to rise in cardiogenic shock.
Transfusion, repositioning, and ice baths are not indicated interventions.

7. An ED nurse has just received a call from EMS that they are transporting a 17-year-old man who has just sustained a spinal cord injury
(SCI). The nurse recognizes that the most common cause of this type of injury is what?
A) Sports-related injuries
B) Acts of violence
C) Injuries due to a fall
D) Motor vehicle accidents
Ans: D
The most common causes of SCIs are motor vehicle crashes (46%), falls (22%), violence (16%), and sports (12%).

8. A patient with spinal cord injury has a nursing diagnosis of altered mobility and the nurse recognizes the increased the risk of deep vein
thrombosis (DVT). Which of the following would be included as an appropriate nursing intervention to prevent a DVT from occurring?
A) Placing the patient on a fluid restriction as ordered
B) Applying thigh-high elastic stockings
C) Administering an antifibrinolyic agent
D) Assisting the patient with passive range of motion (PROM) exercises
Ans: B
It is important to promote venous return to the heart and prevent venous stasis in a patient with altered mobility. Applying elastic stockings
will aid in the prevention of a DVT. The patient should not be placed on fluid restriction because a dehydrated state will increase the risk of
clotting throughout the body. Antifibrinolytic agents cause the blood to clot, which is absolutely contraindicated in this situation. PROM
exercises are not an effective protection against the development of DVT.

9. Paramedics have brought an intubated patient to the RD following a head injury due to acceleration- deceleration motor vehicle accident.
Increased ICP is suspected. Appropriate nursing interventions would include which of the following?
A) Keep the head of the bed (HOB) flat at all times.
B) Teach the patient to perform the Valsalva maneuver.
C) Administer benzodiazepines on a PRN basis.
D) Perform endotracheal suctioning every hour.
Ans: C
If the patient with a brain injury is very agitated, benzodiazepines are the most commonly used sedatives and do not affect cerebral blood
flow or ICP. The HOB should be elevated 30 degrees. Suctioning should be done a limited basis, due to increasing the pressure in the
cranium. The Valsalva maneuver is to be avoided. This also causes increased ICP.

10. A pt who has sustained nondepressed skull fracture is admitted to acute medical unit. Nursing care should include which of the following?
A) Preparation for emergency craniotomy
B) Watchful waiting and close monitoring
C) Administration of inotropic drugs
D) Fluid resuscitation
Ans: B
Nondepressed skull fractures generally do not require surgical treatment; however, close observation of the patient is essential. A craniotomy
would not likely be needed if the fracture is nondepressed. Even if tx is warranted, it is unlikely to include inotropes or fluid resuscitation.

11. A patient who suffered a spinal cord injury is experiencing an exaggerated autonomic response. What aspect of the patients current health
status is most likely to have precipitated this event?
A) The patient received a blood transfusion.
B) The patients analgesia regimen was recent changed.
C) The patient was not repositioned during the night shift.
D) The patients urinary catheter became occluded.
Ans: D
A distended bladder is the most common cause of autonomic dysreflexia. Infrequent positioning is a less likely cause, although pressure
ulcers or tactile stimulation can cause it. Changes in mediations or blood transfusions are unlikely causes.

12. A patient is admitted to the neurologic ICU with a spinal cord injury. In writing the patients care plan, the nurse specifies that contractures
can best be prevented by what action?
A) Repositioning the patient every 2 hours
B) Initiating range-of-motion exercises (ROM) as soon as the patient initiates
C) Initiating (ROM) exercises as soon as possible after the injury
D) Performing ROM exercises once a day
Ans: C
Passive ROM exercises should be implemented as soon as possible after injury. It would be inappropriate to wait for the patient to first
initiate exercises. Toes, metatarsals, ankles, knees, and hips should be put through a full ROM at least four, and ideally five, times daily.
Repositioning alone will not prevent contractures.
13. A patient with a head injury has been increasingly agitated and the nurse has consequently identified a risk for injury. What is the nurses
best intervention for preventing injury?
A) Restrain the patient as ordered.
B) Administer opioids PRN as ordered.
C) Arrange for friends and family members to sit with the patient.
D) Pad the side rails of the patients bed.
Ans: D
To protect the patient from self-injury, the nurse uses padded side rails. The nurse should avoid restraints, because straining against them can
increase ICP or cause other injury. Narcotics used to control restless patients should be avoided because these medications can depress
respiration, constrict the pupils, and alter the patients responsiveness. Visitors should be limited if the patient is agitated.

14. A pt w/C5 spinal cord injury is tetraplegic. After being moved out of ICU, the pt C/O severe throbbing HA. What should the nurse do 1st?
A) Check the patients indwelling urinary catheter for kinks to ensure patency.
B) Lower the HOB to improve perfusion.
C) Administer analgesia.
D) Reassure the patient that headaches are expected after spinal cord injuries.
Ans: A
A severe throbbing headache is a common symptom of autonomic dysreflexia, which occurs after
injuries to the spinal cord above T6. The syndrome is usually brought on by sympathetic stimulation, such as bowel and bladder distention.
Lowering the HOB can increase ICP. Before administering analgesia, the nurse should check the patients catheter, record vital signs, and
perform an abdominal assessment. A severe throbbing headache is a dangerous symptom in this patient and is not expected.

15. A patient is admitted to the neurologic ICU with a spinal cord injury. When assessing the patient the nurse notes there is a sudden
depression of reflex activity in the spinal cord below the level of injury. What should the nurse suspect?
A) Epidural hemorrhage
B) Hypertensive emergency
C) Spinal shock
D) Hypovolemia
Ans: C
In spinal shock, the reflexes are absent, BP and heart rate fall, and respiratory failure can occur. Hypovolemia, hemorrhage, and hypertension
do not cause this sudden change in neurologic function.

16. An elderly woman found with a head injury on the floor of her home is subsequently admitted to the neurologic ICU. What is the best
rationale for the following physician orders: elevate the HOB; keep the head in neutral alignment with no neck flexion or head rotation; avoid
sharp hip flexion?
A) To decrease cerebral arterial pressure
B) To avoid impeding venous outflow
C) To prevent flexion contractures
D) To prevent aspiration of stomach contents
Ans: B
Any activity or position that impedes venous outflow from the head may contribute to increased volume
inside the skull and possibly increase ICP. Cerebral arterial pressure will be affected by the balance
between oxygen and carbon dioxide. Flexion contractures are not a priority at this time. Stomach
contents could still be aspirated in this position.

17. A patient with a T2 injury is in spinal shock. The nurse will expect to observe what assessment finding?
A) Absence of reflexes along with flaccid extremities
B) Positive Babinskis reflex along with spastic extremities
C) Hyperreflexia along with spastic extremities
D) Spasticity of all four extremities
Ans: A
During the period immediately following a spinal cord injury, spinal shock occurs. In spinal shock, all reflexes are absent and the extremities
are flaccid. When spinal shock subsides, the patient demonstrates a positive Babinskis reflex, hyperreflexia, and spasticity of all four
extremities.

18. A nurse is reviewing the trend of a patients scores on the Glasgow Coma Scale (GCS). This allows the nurse to gauge what aspect of the
patients status?
A) Reflex activity
B) Level of consciousness
C) Cognitive ability
D) Sensory involvement
Ans: B
The Glasgow Coma Scale (GCS) examines three responses related to LOC: eye opening, best verbal response, and best motor response.

19. The nurse is caring for a patient who is rapidly progressing toward brain death. The nurse should be aware of what cardinal signs of brain
death? Select all that apply.
A) Absence of pain response
B) Apnea
C) Coma
D) Absence of brain stem reflexes
E) Absence of deep tendon reflexes
Ans: B, C, D
The three cardinal signs of brain death upon clinical examination are coma, the absence of brain stem reflexes, and apnea. Absences of pain
response and deep tendon reflexes are not necessarily indicative of brain death.

20. Following a spinal cord injury a patient is placed in halo traction. While performing pin site care, the nurse notes that one of the traction
pins has become detached. The nurse would be correct in implementing what priority nursing action?
A) Complete the pin site care to decrease risk of infection.
B) Notify the neurosurgeon of the occurrence.
C) Stabilize the head in a lateral position.
D) Reattach the pin to prevent further head trauma.
Ans: B
If one of the pins became detached, the head is stabilized in neutral position by one person while another notifies the neurosurgeon.
Reattaching the pin as a nursing intervention would not be done due to risk of increased injury. Pin site care would not be a priority in this
instance. Prevention of neurologic injury is the priority.

21. The ED is notified that a 6-year-old is in transit with a suspected brain injury after being struck by a car. The child is unresponsive at this
time, but vital signs are within acceptable limits. What will be the primary goal of initial therapy?
A) Promoting adequate circulation
B) Treating the childs increased ICP
C) Assessing secondary brain injury
D) Preserving brain homeostasis
Ans: D
All therapy is directed toward preserving brain homeostasis and preventing secondary brain injury, which is injury to the brain that occurs
after the original traumatic event. The scenario does not indicate the child has increased ICP or a secondary brain injury at this point.
Promoting circulation is likely secondary to the broader goal of preserving brain homeostasis.

22. A patient is admitted to the neurologic ICU with a suspected diffuse axonal injury. What would be the primary neuroimaging diagnostic
tool used on this patient to evaluate the brain structure?
A) MRI
B) PET scan
C) X-ray
D) Ultrasound
Ans: A
CT and MRI scans, the primary neuroimaging diagnostic tools, are useful in evaluating the brain structure. Ultrasound would not show the
brain nor would an x-ray. A PET scan shows brain function, not brain structure.

23. A 13-year-old was brought to the ED, unconscious, after being hit in the head by a baseball. When the child regains consciousness, 5
hours after being admitted, he cannot remember the traumatic event. MRI shows no structural sign of injury. What injury would the nurse
suspect the patient has?
A) Diffuse axonal injury
B) Grade 1 concussion with frontal lobe involvement
C) Contusion
D) Grade 3 concussion with temporal lobe involvement
Ans: D
In a grade 3 concussion there is a loss of consciousness lasting from seconds to minutes. Temporal lobe involvement results in amnesia.
Frontal lobe involvement can cause uncharacteristic behavior and a grade 1 concussion does not involve loss of consciousness. Diagnostic
studies may show no apparent structural sign of injury, but the duration of unconsciousness is an indicator of the severity of the
concussion. Diffuse axonal injury (DAI) results from widespread shearing and rotational forces that produce damage throughout the brainto
axons in the cerebral hemispheres, corpus callosum, and brain stem. In cerebral contusion, a moderate to severe head injury, the brain is
bruised and damaged in a specific area because of severe acceleration-deceleration force or blunt trauma.

24. An 82-year-old man is admitted for observation after a fall. Due to his age, the nurse knows that the patient is at increased risk for what
complication of his injury?
A) Hematoma
B) Skull fracture
C) Embolus
D) Stroke
Ans: A
Two major factors place older adults at increased risk for hematomas. First, the dura becomes more adherent to the skull with increasing age.
Second, many older adults take aspirin and anticoagulants as part of routine management of chronic conditions. Because of these factors, the
patients risk of a hematoma is likely greater than that of stroke, embolism, or skull fracture.
25. A neurologic flow chart is often used to document the care of a patient with a traumatic brain injury. At what point in the patients care
should the nurse begin to use a neurologic flow chart?
A) When the patients condition begins to deteriorate
B) As soon as the initial assessment is made
C) At the beginning of each shift
D) When there is a clinically significant change in the patients condition
Ans: B
Neurologic parameters are assessed initially and as frequently as the patients condition requires. As soon as the initial assessment is made, the
use of a neurologic flowchart is started and maintained. A new chart is not begun at the start of every shift.

26. The nurse planning the care of a patient with head injuries is addressing the patients nursing diagnosis of sleep deprivation. What action
should the nurse implement?
A) Administer a benzodiazepine at bedtime each night.
B) Do not disturb the patient between 2200 and 0600.
C) Cluster overnight nursing activities to minimize disturbances.
D) Ensure that the patient does not sleep during the day.
Ans: C
To allow the patient longer times of uninterrupted sleep and rest, the nurse can group nursing care activities so that the patient is disturbed
less frequently. However, it is impractical and unsafe to provide no care for an 8-hour period. The use of benzodiazepines should be avoided.

27. The nurse has implemented interventions aimed at facilitating family coping in the care of a patient with a traumatic brain injury. How
can the nurse best facilitate family coping?
A) Help the family understand that the patient could have died.
B) Emphasize the importance of accepting the patients new limitations.
C) Have the members of the family plan the patients inpatient care.
D) Assist the family in setting appropriate short-term goals.
Ans: D
Helpful interventions to facilitate coping include providing family members with accurate and honest information and encouraging them to
continue to set well-defined, short-term goals. Stating that a patients condition could be worse downplays their concerns. Emphasizing the
importance of acceptance may not necessarily help the family accept the patients condition. Family members cannot normally plan a patients
hospital care, although they may contribute to the care in some ways.

28. The school nurse is giving a presentation on preventing spinal cord injuries (SCI). What should the nurse identify as prominent risk
factors for SCI? Select all that apply.
A) Young age
B) Frequent travel
C) African American race
D) Male gender
E) Alcohol or drug use
Ans: A, D, E
The predominant risk factors for SCI include young age, male gender, and alcohol and drug use. Ethnicity and travel are not risk factors.

29. The school nurse has been called to the football field where player is immobile on the field after landing awkwardly on his head during a
play. While awaiting an ambulance, what action should the nurse perform?
A) Ensure that the player is not moved.
B) Obtain the players vital signs, if possible.
C) Perform a rapid assessment of the players range of motion.
D) Assess the players reflexes.
Ans: A
At the scene of the injury, the patient must be immobilized on a spinal (back) board, with the head and neck maintained in a neutral position,
to prevent an incomplete injury from becoming complete. This is a priority over determining the patients vital signs. It would be
inappropriate to test ROM or reflexes.

30. The nurse is caring for a patient whose spinal cord injury has caused recent muscle spasticity. What medication should the nurse expect to
be ordered to control this?
A) Baclofen (Lioresal)
B) Dexamethasone (Decadron)
C) Mannitol (Osmitrol)
D) Phenobarbital (Luminal)
Ans: A
Baclofen is classified as an antispasmodic agent in the treatment of muscles spasms related to spinal cord injury. Decadron is an anti-
inflammatory medication used to decrease inflammation in both SCI and head injury. Mannitol is used to decrease cerebral edema in patients
with head injury. Phenobarbital is an anticonvulsant that is used in the treatment of seizure activity.
31. The nurse is planning the care of a patient with a T1 spinal cord injury. The nurse has identified the diagnosis of risk for impaired skin
integrity. How can the nurse best address this risk?
A) Change the patients position frequently.
B) Provide a high-protein diet.
C) Provide light massage at least daily.
D) Teach the patient deep breathing and coughing exercises.
Ans: A
Frequent position changes are among the best preventative measures against pressure ulcers. A high- protein diet can benefit wound healing,
but does not necessarily prevent skin breakdown. Light massage and deep breathing do not protect or restore skin integrity.

32. A patient with a spinal cord injury has experienced several hypotensive episodes. How can the nurse best address the patients risk for
orthostatic hypotension?
A) Administer an IV bolus of normal saline prior to repositioning.
B) Maintain bed rest until normal BP regulation returns.
C) Monitor the patients BP before and during position changes.
D) Allow the patient to initiate repositioning.
Ans: C
To prevent hypotensive episodes, close monitoring of vital signs before & during position changes is essential. Prolonged bed rest carries
numerous risks and it is not possible to provide a bolus before each position change. Following the pts lead may or may not help regulate BP.

33. A nurse on the neurologic unit is providing care for a patient who has spinal cord injury at the level of C4. When planning the patients
care, what aspect of the patients neurologic and functional status should the nurse consider?
A) The patient will be unable to use a wheelchair.
B) The patient will be unable to swallow food.
C) The patient will be continent of urine, but incontinent of bowel.
D) The patient will require full assistance for all aspects of elimination.
Ans: D
Patients with a lesion at C4 are fully dependent for elimination. The patient is dependent for feeding, but is able to swallow. The patient will
be capable of using an electric wheelchair.

34. The nurse is providing health education to a patient who has a C6 spinal cord injury. The patient asks why autonomic dysreflexia is
considered an emergency. What would be the nurses best answer?
A) The sudden increase in BP can raise the ICP or rupture a cerebral blood vessel.
B) The suddenness of the onset of the syndrome tells us the body is struggling to maintain its normal state.
C) Autonomic dysreflexia causes permanent damage to delicate nerve fibers that are healing.
D) The sudden, severe headache increases muscle tone and can cause further nerve damage.
Ans: A
The sudden increase in BP may cause a rupture of one or more cerebral blood vessels or lead to increased ICP. Autonomic dysreflexia does
not directly cause nerve damage.

35. The nurse caring for a patient with a spinal cord injury notes that the patient is exhibiting early signs and symptoms of disuse syndrome.
Which of the following is the most appropriate nursing action?
A) Limit the amount of assistance provided with ADLs.
B) Collaborate with the physical therapist and immobilize the patients extremities temporarily.
C) Increase the frequency of ROM exercises.
D) Educate the patient about the importance of frequent position changes.
Ans: C
To prevent disuse syndrome, ROM exercises must be provided at least four times a day, and care is taken to stretch the Achilles tendon with
exercises. The patient is repositioned frequently and is maintained in proper body alignment whether in bed or in a wheelchair. The patient
must be repositioned by caregivers, not just taught about repositioning. It is inappropriate to limit assistance for the sole purpose of
preventing disuse syndrome.

36. Splints have been ordered for a patient who is at risk of developing footdrop following a spinal cord injury. The nurse caring for this
patient knows that the splints are removed and reapplied when?
A) At the patients request
B) Each morning and evening
C) Every 2 hours
D) One hour prior to mobility exercises
Ans: C
The feet are prone to footdrop; therefore, various types of splints are used to prevent footdrop. When used, the splints are removed and
reapplied every 2 hours.

37. A patient who is being treated in the hospital for a spinal cord injury is advocating for the removal of his urinary catheter, stating that he
wants to try to resume normal elimination. What principle should guide the care teams decision regarding this intervention?
A) Urinary retention can have serious consequences in patients with SCIs.
B) Urinary function is permanently lost following an SCI.
C) Urinary catheters should not remain in place for more than 7 days.
D) Overuse of urinary catheters can exacerbate nerve damage.
Ans: A
Bladder distention, a major cause of autonomic dysreflexia, can also cause trauma. For this reason, removal of a urinary catheter must be
considered with caution. Extended use of urinary catheterization is often necessary following SCI. The effect of a spinal cord lesion on
urinary function depends on the level of the injury. Catheter use does not cause nerve damage, although it is a major risk factor for UTIs.

38. A patient with spinal cord injury is ready to be discharged home. A family member asks the nurse to review potential complications one
more time. What are the potential complications that should be monitored for in this patient? Select all that apply.
A) Orthostatic hypotension
B) Autonomic dysreflexia
C) DVT
D) Salt-wasting syndrome
E) Increased ICP
Ans: A, B, C
For a spinal cord-injured patient, based on the assessment data, potential complications that may develop include DVT, orthostatic
hypotension, and autonomic dysreflexia. Salt-wasting syndrome or increased ICP are not typical complications following the immediate
recovery period.

39. The nurse recognizes that a patient with a SCI is at risk for muscle spasticity. How can the nurse best prevent this complication of an SCI?
A) Position the patient in a high Fowlers position when in bed.
B) Support the knees with a pillow when the patient is in bed.
C) Perform passive ROM exercises as ordered.
D) Administer NSAIDs as ordered.
Ans: C
Passive ROM exercises can prevent muscle spasticity following SCI. NSAIDs are not used for this purpose. Pillows and sitting upright do not
directly address the patients risk of muscle spasticity.

40. A patient is admitted to the neurologic ICU with a C4 spinal cord injury. When writing the plan of care for this patient, which of the
following nursing diagnoses would the nurse prioritize in the immediate care of this patient?
A) Risk for impaired skin integrity related to immobility and sensory loss
B) Impaired physical mobility related to loss of motor function
C) Ineffective breathing patterns related to weakness of the intercostal muscles
D) Urinary retention related to inability to void spontaneously
Ans: C
A nursing diagnosis related to breathing pattern would be the priority for this patient. A C4 spinal cord injury will require ventilatory support,
due to the diaphragm and intercostals being affected. The other nursing diagnoses would be used in the care plan, but not designated as a
higher priority than ineffective breathing patterns.
Chapter 69: Management of Patients with Neurologic Infections,
Autoimmune Disorders, and Neuropathies
1. A patient with possible bacterial meningitis is admitted to the ICU. What assessment finding would the
nurse expect for a patient with this diagnosis?
A) Pain upon ankle dorsiflexion of the foot
B) Neck flexion produces flexion of knees and hips
C) Inability to stand with eyes closed and arms extended without swaying
D) Numbness and tingling in the lower extremities
Ans: B
Clinical manifestations of bacterial meningitis include a positive Brudzinskis sign. Neck flexion producing flexion of knees & hips correlates
with a positive Brudzinskis sign. Positive Homans sign (pain upon dorsiflexion of the foot) and negative Rombergs sign (inability to stand
with eyes closed and arms extended) are not expected assessment findings for the patient with bacterial meningitis. Peripheral neuropathy
manifests as numbness and tingling in the lower extremities. Again, this would not be an initial assessment to rule out bacterial meningitis.

2. The nurse is planning discharge education for a patient with trigeminal neuralgia. The nurse knows to include information about factors
that precipitate an attack. What would the nurse be correct in teaching the patient to avoid?
A) Washing his face
B) Exposing his skin to sunlight
C) Using artificial tears
D) Drinking large amounts of fluids
Ans: A
Washing the face should be avoided if possible because this activity can trigger an attack of pain in a patient with trigeminal neuralgia. Using
artificial tears would be an appropriate behavior. Exposing the skin to sunlight would not be harmful to this patient. Temperature extremes in
beverages should be avoided.

3. The nurse is caring for a patient with multiple sclerosis (MS). The patient tells the nurse the hardest thing to deal with is the fatigue. When
teaching the patient how to reduce fatigue, what action should the nurse suggest?
A) Taking a hot bath at least once daily
B) Resting in an air-conditioned room whenever possible
C) Increasing the dose of muscle relaxants
D) Avoiding naps during the day
Ans: B
Fatigue is a common symptom of patients with MS. Lowering the body temperature by resting in an air- conditioned room may relieve
fatigue; however, extreme cold should be avoided. A hot bath or shower can increase body temperature, producing fatigue. Muscle relaxants,
prescribed to reduce spasticity, can cause drowsiness and fatigue. Planning for frequent rest periods and naps can relieve fatigue. Other
measures to reduce fatigue in the patient with MS include treating depression, using occupational therapy to learn energy conservation
techniques, and reducing spasticity.

4. A patient with Guillain-Barr syndrome has experienced a sharp decline in vital capacity. What is the nurses most appropriate action?
A) Administer bronchodilators as ordered.
B) Remind the patient of the importance of deep breathing and coughing exercises.
C) Prepare to assist with intubation.
D) Administer supplementary oxygen by nasal cannula.
Ans: C
For the patient with Guillain-Barr syndrome, mechanical ventilation is required if the vital capacity falls, making spontaneous breathing
impossible and tissue oxygenation inadequate. Each of the other listed actions is likely insufficient to meet the patients oxygenation needs.

5. A patient diagnosed with Bells palsy is being cared for on an outpatient basis. During health education, the nurse should promote which of
the following actions?
A) Applying a protective eye shield at night
B) Chewing on the affected side to prevent unilateral neglect
C) Avoiding the use of analgesics whenever possible
D) Avoiding brushing the teeth
Ans: A
Corneal irritation and ulceration may occur if the eye is unprotected. While paralysis lasts, the involved eye must be protected. The patient
should be encouraged to eat on the unaffected side, due to swallowing difficulties. Analgesics are used to control the facial pain. The patient
should continue to provide self-care including oral hygiene.

6. The nurse is working with a patient who is newly diagnosed with MS. What basic information should the nurse provide to the patient?
A) MS is a progressive demyelinating disease of the nervous system.
B) MS usually occurs more frequently in men.
C) MS typically has an acute onset.
D) MS is sometimes caused by a bacterial infection.
Ans: A
MS is a chronic, degenerative, progressive disease of the central nervous system, characterized by the occurrence of small patches of
demyelination in the brain and spinal cord. The cause of MS is not known, and the disease affects twice as many women as men.

7. The nurse is creating a plan of care for a patient who has a recent diagnosis of MS. Which of the following should the nurse include in the
patients care plan?
A) Encourage patient to void every hour.
B) Order a low-residue diet.
C) Provide total assistance with all ADLs.
D) Instruct the patient on daily muscle stretching.
Ans: D
A patient diagnosed with MS should be encouraged to increase the fiber in his or her diet and void 30 minutes after drinking to help train the
bladder. The patient should participate in daily muscle stretching to help alleviate and relax muscle spasms.

8. A patient with metastatic cancer has developed trigeminal neuralgia and is taking carbamazepine (Tegretol) for pain relief. What principle
applies to the administration of this medication?
A) Tegretol is not known to have serious adverse effects.
B) The patient should be monitored for bone marrow depression.
C) Side effects of the medication include renal dysfunction.
D) The medication should be first taken in the maximum dosage form to be effective.
Ans: B
The anticonvulsant agents carbamazepine (Tegretol) and phenytoin (Dilantin) relieve pain in most patients diagnosed with trigeminal
neuralgia by reducing the transmission of impulses at certain nerve terminals. Side effects include nausea, dizziness, drowsiness, and aplastic
anemia. Carbamazepine should be gradually increased until pain relief is obtained.

9. A male patient presents to the clinic complaining of a headache. The nurse notes that the patient is guarding his neck and tells the nurse that
he has stiffness in the neck area. The nurse suspects the patient may have meningitis. What is another well-recognized sign of this infection?
A) Negative Brudzinskis sign
B) Positive Kernigs sign
C) Hyperpatellar reflex
D) Sluggish pupil reaction
Ans: B
Meningeal irritation results in a number of well-recognized signs commonly seen in meningitis, such as a positive Kernigs sign, a positive
Brudzinskis sign, and photophobia. Hyperpatellar reflex and a sluggish pupil reaction are not commonly recognized signs of meningitis.

10. The nurse is developing a plan of care for a patient newly diagnosed with Bells palsy. The nurses plan of care should address what
characteristic manifestation of this disease?
A) Tinnitus
B) Facial paralysis
C) Pain at the base of the tongue
D) Diplopia
Ans: B
Bells palsy is characterized by facial dysfunction, weakness, and paralysis. It doesn't result in diplopia, pain at the base of tongue, or tinnitus.

11. The nurse caring for a patient diagnosed with Guillain-Barr syndrome is planning care with regard to the clinical manifestations
associated this syndrome. The nurses communication with the patient should reflect the possibility of what sign or symptom of the disease?
A) Intermittent hearing loss
B) Tinnitus
C) Tongue enlargement
D) Vocal paralysis
Ans: D
Guillain-Barr syndrome is a disorder of the vagus nerve. Clinical manifestations include vocal paralysis, dysphagia, and voice changes
(temporary or permanent hoarseness). Hearing deficits, tinnitus, and tongue enlargement are not associated with the disease.

12. The nurse is preparing to provide care for a patient diagnosed with myasthenia gravis. The nurse should know that the signs and
symptoms of the disease are the result of what?
A) Genetic dysfunction
B) Upper and lower motor neuron lesions
C) Decreased conduction of impulses in an upper motor neuron lesion
D) A lower motor neuron lesion
Ans: D
Myasthenia gravis is characterized by a weakness of muscles, especially in the face and throat, caused by a lower neuron lesion at the
myoneural junction. It is not a genetic disorder. A combined upper and lower neuron lesion generally occurs as a result of spinal injuries. A
lesion involving cranial nerves and their axons in the spinal cord would cause decreased conduction of impulses at an upper motor neuron.
13. A patient with suspected Creutzfeldt-Jakob disease (CJD) is being admitted to the unit. The nurse would expect what diagnostic test to be
ordered for this patient?
A) Cerebral angiography
B) ABG analysis
C) CT
D) EEG
Ans: D
The EEG reveals a characteristic pattern over the duration of CJD. A CT scan may be used to rule out disorders that may mimic the symptoms
of CJD. ABGs would not be necessary until the later stages of CJD; they would not be utilized as a diagnostic test. Cerebral angiography is
not used to diagnose CJD.

14. To alleviate pain associated with trigeminal neuralgia, a patient is taking Tegretol (carbamazepine). What health education should the
nurse provide to the patient before initiating this treatment?
A) Concurrent use of calcium supplements is contraindicated.
B) Blood levels of the drug must be monitored.
C) The drug is likely to cause hyperactivity and agitation.
D) Tegretol can cause tinnitus during the first few days of treatment.
Ans: B
Side effects of Tegretol include nausea, dizziness, drowsiness, and aplastic anemia. The patient must also be monitored for bone marrow
depression during long-term therapy. Skin discoloration, insomnia, and tinnitus are not side effects of Tegretol.

15. A patient with herpes simplex virus encephalitis (HSV) has been admitted to the ICU. What medication would the nurse expect the
physician to order for the treatment of this disease process?
A) Cyclosporine (Neoral)
B) Acyclovir (Zovirax)
C) Cyclobenzaprine (Flexeril)
D) Ampicillin (Prinicpen)
Ans: B
Feedback:
Acyclovir (Zovirax) or ganciclovir (Cytovene), antiviral agents, are the medications of choice in the treatment of HSV. The mode of action is
the inhibition of viral DNA replication. To prevent relapse, treatment would continue for up to 3 weeks. Cyclosporine is an
immunosuppressant and antirheumatic. Cyclobenzaprine is a centrally acting skeletal muscle relaxant. Ampicillin, an antibiotic, is ineffective
against viruses.

16. A middle-aged woman has sought care from her primary care provider and undergone diagnostic testing that has resulted in a diagnosis of
MS. What sign or symptom is most likely to have prompted the woman to seek care?
A) Cognitive declines
B) Personality changes
C) Contractures
D) Difficulty in coordination
Ans: D
The primary symptoms of MS most commonly reported are fatigue, depression, weakness, numbness, difficulty in coordination, loss of
balance, spasticity, and pain. Cognitive changes and contractures usually occur later in the disease.

17. A nurse is planning the care of a 28-year-old woman hospitalized with a diagnosis of myasthenia gravis. What approach would be most
appropriate for the care and scheduling of diagnostic procedures for this patient?
A) All at one time, to provide a longer rest period
B) Before meals, to stimulate her appetite
C) In the morning, with frequent rest periods
D) Before bedtime, to promote rest
Ans: C
Procedures should be spaced to allow for rest in between. Procedures should be avoided before meals, or the patient may be too exhausted to
eat. Procedures should be avoided near bedtime if possible.

18. The nurse is caring for a patient who is hospitalized with an exacerbation of MS. To ensure the patients safety, what nursing action should
be performed?
A) Ensure that suction apparatus is set up at the bedside.
B) Pad the patients bed rails.
C) Maintain bed rest whenever possible.
D) Provide several small meals each day.
Ans: A
Because of the patients risk of aspiration, it is important to have a suction apparatus at hand. Bed rest should be generally be minimized, not
maximized, and there is no need to pad the patients bed rails or to provide multiple small meals.

19. A 33-year-old patient presents at the clinic with complaints of weakness, incoordination, dizziness, and loss of balance. The patient is
hospitalized and diagnosed with MS. What sign or symptom, revealed during the initial assessment, is typical of MS?
A) Diplopia, history of increased fatigue, and decreased or absent deep tendon reflexes
B) Flexor spasm, clonus, and negative Babinskis reflex
C) Blurred vision, intention tremor, and urinary hesitancy
D) Hyperactive abdominal reflexes and history of unsteady gait and episodic paresthesia in both legs
Ans: C
Optic neuritis, leading to blurred vision, is a common early sign of MS, as is intention tremor (tremor when performing an activity). Nerve
damage can cause urinary hesitancy. In MS, deep tendon reflexes are increased or hyperactive. A positive Babinskis reflex is found in MS.
Abdominal reflexes are absent with MS.

20. The nurse is developing a plan of care for a patient with Guillain-Barr syndrome. Which of the following interventions should the nurse
prioritize for this patient?
A) Using the incentive spirometer as prescribed
B) Maintaining the patient on bed rest
C) Providing aids to compensate for loss of vision
D) Assessing frequently for loss of cognitive function
Ans: A
Respiratory function can be maximized with incentive spirometry and chest physiotherapy. Nursing interventions toward enhancing physical
mobility should be utilized. Nursing interventions are aimed at preventing a deep vein thrombosis. Guillain-Barr syndrome does not affect
cognitive function or vision.

21. A 69-year-old patient is brought to the ED by ambulance because a family member found him lying on the floor disoriented and lethargic.
The physician suspects bacterial meningitis and admits the patient to the ICU. The nurse knows that risk factors for an unfavorable outcome
include what? Select all that apply.
A) Blood pressure greater than 140/90 mm Hg
B) Heart rate greater than 120 bpm
C) Older age
D) Low Glasgow Coma Scale
E) Lack of previous immunizations
Ans: B, C, D
Risks for an unfavorable outcome of meningitis include older age, a heart rate greater than 120 beats/minute, low Glasgow Coma Scale score,
cranial nerve palsies, and a positive Gram stain 1 hour after presentation to the hospital. A BP greater than 140/90 mm Hg is indicative of
hypertension, but is not necessarily related to poor outcomes related to meningitis. Immunizations are not normally relevant
to the course of the disease.

22. The critical care nurse is caring for 25-year-old man admitted to the ICU with a brain abscess. What is a priority nursing responsibility in
the care of this patient?
A) Maintaining the patients functional independence
B) Providing health education
C) Monitoring neurologic status closely
D) Promoting mobility
Ans: C
Vigilant neurologic monitoring is a key aspect of caring for a patient who has a brain abscess. This supersedes education, ADLs, and mobility,
even though these are all valid and important aspects of nursing care.

23. A patient is being admitted to the neurologic ICU with suspected herpes simplex virus encephalitis. What nursing action best addresses
the patients complaints of headache?
A) Initiating a patient-controlled analgesia (PCA) of morphine sulfate
B) Administering hydromorphone (Dilaudid) IV as needed
C) Dimming the lights and reducing stimulation
D) Distracting the patient with activity
Ans: C
Comfort measures to reduce headache include dimming the lights, limiting noise and visitors, grouping nursing interventions, and
administering analgesic agents. Opioid analgesic medications may mask neurologic symptoms; therefore, they are used cautiously. Non-
opioid analgesics may be preferred. Distraction is unlikely to be effective, and may exacerbate the patients pain.

24. A patient is admitted through the ED with suspected St. Louis encephalitis. The unique clinical feature of St. Louis encephalitis will make
what nursing action a priority?
A) Serial assessments of hemoglobin levels
B) Blood glucose monitoring
C) Close monitoring of fluid balance
D) Assessment of pain along dermatomes
Ans: C
A unique clinical feature of St. Louis encephalitis is SIADH with hyponatremia. As such, it is important to monitor the patients intake and
output closely.
25. The nurse is caring for a 77-year-old woman with MS. She states that she is very concerned about the progress of her disease and what the
future holds. The nurse should know that elderly patients with MS are known to be particularly concerned about what variables? Select all
that apply.
A) Possible nursing home placement
B) Pain associated with physical therapy
C) Increasing disability
D) Becoming a burden on the family
E) Loss of appetite
Ans: A, C, D
Elderly patients with MS are particularly concerned about increasing disability, family burden, marital concern, and the possible future need
for nursing home care. Older adults with MS are not noted to have particular concerns regarding the pain of therapy or loss of appetite.

26. You are the clinic nurse caring for a patient with a recent diagnosis of myasthenia gravis. The patient has begun treatment with
pyridostigmine bromide (Mestinon). What change in status would most clearly suggest a therapeutic benefit of this medication?
A) Increased muscle strength
B) Decreased pain
C) Improved GI function
D) Improved cognition
Ans: A
The goal of treatment using pyridostigmine bromide is improvement of muscle strength and control of fatigue. The drug is not intended to
treat pain, or cognitive or GI functions.

27. The critical care nurse is admitting a patient in myasthenic crisis to the ICU. The nurse should prioritize what nursing action in the
immediate care of this patient?
A) Suctioning secretions
B) Facilitating ABG analysis
C) Providing ventilatory assistance
D) Administering tube feedings
Ans: C
Providing ventilatory assistance takes precedence in the immediate management of the patient with myasthenic crisis. It may be necessary to
suction secretions and/or provide tube feedings, but they are not the priority for this patient. ABG analysis will be done, but this is not the
priority.

28. The nurse caring for a pt in ICU diagnosed with Guillain-Barr syndrome should prioritize monitoring for what potential complication?
A) Impaired skin integrity
B) Cognitive deficits
C) Hemorrhage
D) Autonomic dysfunction
Ans: D
Based on the assessment data, potential complications that may develop include respiratory failure and autonomic dysfunction. Skin
breakdown, decreased cognition, and hemorrhage are not complications of Guillain-Barr syndrome.

29. The nurse is teaching a patient with Guillain-Barr syndrome about the disease. The patient asks how he can ever recover if demyelination
of his nerves is occurring. What would be the nurses best response?
A) Guillain-Barr spares the Schwann cell, which allows for remyelination in the recovery phase of the disease.
B) In Guillain-Barr, Schwann cells replicate themselves before the disease destroys them, so remyelination is possible.
C) I know you understand that nerve cells do not remyelinate, so the physician is the best one to answer your question.
D) For some reason, in Guillain-Barr, Schwann cells become activated and take over the remyelination process.
Ans: A
Myelin is a complex substance that covers nerves, providing insulation and speeding the conduction of impulses from the cell body to the
dendrites. The cell that produces myelin in the peripheral nervous system is the Schwann cell. In Guillain-Barr syndrome, the Schwann cell is
spared, allowing for remyelination in the recovery phase of the disease. The nurse should avoid downplaying the patients concerns by wholly
deferring to the physician.

30. A patient diagnosed with myasthenia gravis has been hospitalized to receive plasmapheresis for a myasthenic exacerbation. The nurse
knows that the course of treatment for plasmapheresis in a patient with myasthenia gravis is what?
A) Every day for 1 week
B) Determined by the patients response
C) Alternate days for 10 days
D) Determined by the patients weight
Ans: B
The typical course of plasmapheresis consists of daily or alternate-day treatment, and # of treatments is determined by the patients response.

31. The nurse is discharging a patient home after surgery for trigeminal neuralgia. What advice should the nurse provide to this patient in
order to reduce the risk of injury?
A) Avoid watching television or using a computer for more than 1 hour at a time.
B) Use OTC antibiotic eye drops for at least 14 days.
C) Avoid rubbing the eye on the affected side of the face.
D) Rinse the eye on the affected side with normal saline daily for 1 week.
Ans: C
If the surgery results in sensory deficits to the affected side of the face, the patient is instructed not to rub the eye because the pain of a
resulting injury will not be detected. There is no need to limit TV viewing or to rinse the eye daily. Antibiotics may or may not be prescribed,
and these would not reduce the risk of injury.

32. A patient diagnosed with Bells palsy is having decreased sensitivity to touch of the involved nerve. What should the nurse recommend to
prevent atrophy of the muscles?
A) Blowing up balloons
B) Deliberately frowning
C) Smiling repeatedly
D) Whistling
Ans: D
Facial exercises, such as wrinkling the forehead, blowing out the cheeks, and whistling, may be performed with the aid of a mirror to prevent
muscle atrophy. Blowing up balloons, frowning, and smiling are not considered facial exercises.

33. A pt w/DM presents to the clinic & is diagnosed with a mononeuropathy. This pts nursing care should involve which of the following?
A) Protection of the affected limb from injury
B) Passive and active ROM exercises for the affected limb
C) Education about improvements to glycemic control
D) Interventions to prevent contractures
Ans: A
Nursing care involves protection of the affected limb or area from injury, as well as appropriate patient teaching about mononeuropathy and
its treatment. Nursing care for this patient does not likely involve exercises or assistive devices, since these are unrelated to the etiology of the
disease. Improvements to diabetes management may or may not be necessary.

34. A patient diagnosed with MS has been admitted to the medical unit for treatment of an MS exacerbation. Included in the admission orders
is baclofen (Lioresal). What should the nurse identify as an expected outcome of this treatment?
A) Reduction in the appearance of new lesions on the MRI
B) Decreased muscle spasms in the lower extremities
C) Increased muscle strength in the upper extremities
D) Decreased severity and duration of exacerbations
Ans: B
Baclofen, a g-aminobutyric acid (GABA) agonist, is the medication of choice in treating spasms. It can be administered orally or by
intrathecal injection. Avonex and Betaseron reduce the appearance of new lesions on the MRI. Corticosteroids limit the severity and duration
of exacerbations. Anticholinesterase agents increase muscle strength in the upper extremities.

35. A 35-year-old woman is diagnosed with a peripheral neuropathy. When making her plan of care, the nurse knows to include what in
patient teaching? Select all that apply.
A) Inspect the lower extremities for skin breakdown.
B) Footwear needs to be accurately sized.
C) Immediate family members should be screened for the disease.
D) Assistive devices may be needed to reduce the risk of falls.
E) Dietary modifications are likely necessary.
Ans: A, B, D
The plan of care includes inspection of the lower extremities for skin breakdown. Footwear should be accurately sized. Assistive devices,
such as a walker or cane, may decrease the risk of falls. Bath water temperature is checked to avoid thermal injury. Peripheral neuropathies do
not have a genetic component and diet is unrelated.

36. A 73-year-old man comes to the clinic complaining of weakness and loss of sensation in his feet and legs. Assessment of the patient
shows decreased reflexes bilaterally. Why would it be a challenge to diagnose a peripheral neuropathy in this patient?
A) Older adults are often vague historians.
B) The elderly have fewer peripheral nerves than younger adults.
C) Many older adults are hesitant to admit that their body is changing.
D) Many symptoms can be the result of normal aging process.
Ans: D
The diagnosis of peripheral neuropathy in the geriatric population is challenging because many symptoms, such as decreased reflexes, can be
associated with the normal aging process. In this scenario, the patient has come to the clinic seeking help for his problem; this does not
indicate a desire on the part of the patient to withhold information from the health care giver. The normal aging process does not include a
diminishing number of peripheral nerves.
37. A patient with MS has been admitted to the hospital following an acute exacerbation. When planning the patients care, the nurse addresses
the need to enhance the patients bladder control. What aspect of nursing care is most likely to meet this goal?
A) Establish a timed voiding schedule.
B) Avoid foods that change the pH of urine.
C) Perform intermittent catheterization q6h.
D) Administer anticholinergic drugs as ordered.
Ans: A
A timed voiding schedule addresses many of the challenges with urinary continence that face the patient with MS. Interventions should be
implemented to prevent the need for catheterization and anticholinergics are not normally used.

38. A pt w/MS has developed dysphagia as a result of cranial nerve dysfunction. What nursing action should the nurse consequently perform?
A) Arrange for the patient to receive a low residue diet.
B) Position the patient upright during feeding.
C) Suction the patient following each meal.
D) Withhold liquids until the patient has finished eating.
Ans: B
Correct, upright positioning is necessary to prevent aspiration in the patient with dysphagia. There is no need for a low-residue diet and
suctioning should not be performed unless there is an apparent need. Liquids do not need to be withheld during meals in order to prevent
aspiration.

39. A 48-year-old patient has been diagnosed with trigeminal neuralgia following recent episodes of unilateral face pain. The nurse should
recognize what implication of this diagnosis?
A) The patient will likely require lifelong treatment with anticholinergic medications.
B) The patient has a disproportionate risk of developing myasthenia gravis later in life.
C) The patient needs to be assessed for MS.
D) The disease is self-limiting and the patient will achieve pain relief over time.
Ans: C
Patients that develop trigeminal neuralgia before age 50 should be evaluated for the coexistent of MS because trigeminal neuralgia occurs in
approximately 5% of patients with MS. Treatment does not include anticholinergics and the disease is not self-limiting. Trigeminal neuralgia
is not associated with an increased risk of myasthenia gravis.

40. A patient presents at the clinic complaining of pain and weakness in her hands. On assessment, the nurse notes diminished reflexes in the
upper extremities bilaterally and bilateral loss of sensation. The nurse knows that these findings are indicative of what?
A) Guillain-Barr syndrome
B) Myasthenia gravis
C) Trigeminal neuralgia
D) Peripheral nerve disorder
Ans: D
The major symptoms of peripheral nerve disorders are loss of sensation, muscle atrophy, weakness, diminished reflexes, pain, and paresthesia
(numbness, tingling) of the extremities. Trigeminal neuralgia is a condition of the fifth cranial nerve that is characterized by paroxysms of
pain in the area innervated by any of the three branches, but most commonly the second and third branches of the trigeminal nerve.
Myasthenia gravis, an autoimmune disorder affecting the myoneural junction, is characterized by varying degrees of weakness of the
voluntary muscles. Guillain-Barr syndrome is an autoimmune attack on the peripheral nerve myelin.

You might also like